0% found this document useful (0 votes)
36 views

Collection of Test Tasks

This section discusses pregnancy, labor, delivery and the postpartum period. It provides 11 multiple choice questions testing knowledge about normal term pregnancy, labor stages, fetal positioning, postpartum examinations and complications.

Uploaded by

Haruna Bzaki
Copyright
© © All Rights Reserved
We take content rights seriously. If you suspect this is your content, claim it here.
Available Formats
Download as PDF, TXT or read online on Scribd
0% found this document useful (0 votes)
36 views

Collection of Test Tasks

This section discusses pregnancy, labor, delivery and the postpartum period. It provides 11 multiple choice questions testing knowledge about normal term pregnancy, labor stages, fetal positioning, postpartum examinations and complications.

Uploaded by

Haruna Bzaki
Copyright
© © All Rights Reserved
We take content rights seriously. If you suspect this is your content, claim it here.
Available Formats
Download as PDF, TXT or read online on Scribd
You are on page 1/ 80

MINISTRY OF HEALTH OF UKRAINE

ZAPORIZHZHIA STATE MEDICAL UNIVERSITY


Department of Obstetrics and Gynecology

Obstetric and Gynecology


Collection of test tasks
on obstetrics and gynecology
for 6th year English-speaking students
of II international faculty of specialty 7.1201000 “General medicine”
to prepare for the licensing exam "Krok 2"

Zaporizhzhia
2018

1
UDC 618.1/2(075.8)
S98
Ratified on meeting of the Central methodical committee
of Zaporizhzhia State Medical University
and it is recommended for the use in educational process for foreign
students(protocol № from )

Compilers:
Yu. Ya. Krut - MD, professor, Head of the Department of Obstetrics and
Gynecology, Zaporizhzhia State Medical University;
V.A.Zaliznyak – PhD, professor of the Department of Obstetrics and
Gynecology, Zaporizhzhia State Medical University;
I. G. Amro – PhD, assistant professor of the Department of Obstetrics and
Gynecology, Zaporizhzhia State Medical University;
O. A. Bohomolova - assistant of the Department of Obstetrics and
Gynecology, Zaporizhzhia State Medical University.

Reviewers:
G. I. Reznichenko – MD, professor, Head of the Department of Obstetrics and
Gynecology SI «Zaporizhia medical academy of post-graduate education
Ministry of health of Ukraine»;
D. Ye. Barkovsky - MD, PhD, DSci., Professor, Department of Obstetric,
Gynecology and Reproductive Medicine of FPE, Zaporizhzhia State Medical
University.

Collection of test tasks on obstetrics and gynecology for 6th year English-
speaking students of II international faculty of specialty 7.1201000 “General
medicine” to prepare for the licensing exam "Krok 2". /: Yu. Ya. Krut,
V.A.Zaliznyak, I. G. Amro, O. A. Bohomolova. – Zaporizhzhia : ZSMU,
2018. – 80 p.

Акушерство та гінекологія. Систематизований збірник тестових завдань


для студентів - іноземних громадян 6 курсу медичних факультетів
спеціальності 7.1201000 «Загальна медицина» для остаточного
модульного контролю КРОК 2 /уклад.: Ю.Я. Круть, В.О.Залізняк, І.Г.
Амро, О.А. Богомолова. – Запоріжжя: ЗДМУ, 2018. – 80 с.

2
СONTENTS

Section I. OBSTETRICS
1. Pregnancy, labor and postpartum period...............................................3
2. Early gestosis of pregnant women. Preeclampsia and eclampsia ..7
3. Premature and postmature pregnancy……………...………………..10
4. Anomalies of the bone pelvis. The problem of macrosomia in modern
obstetrics. Childbirth at malposition and pelvic presentations. Childbirth in
multiple pregnancy………………………………………...…………………13
5. Abnormalities of labor activity……………………...………….……16
6. Fetal hypoxia and feto-placental insufficiency. Immunological conflict
during pregnancy…………………………………………………………......18
7. Bleeding in early pregnancy……………………..…………………..20
8. Obstetrical bleeding………………..………………………………...22
9. Extragenital diseases and pregnancy………………….…………......27
10. Infections during pregnancy.Postpartum infectious complications….31
11. Operative obstetrics………………………………………………….34

Section II. GYNECOLOGY


1. Methods of examination of gynecologic patients…………………...39
2. Inflammatory diseases of female genital organs…………………….42
3. Menstrual disorders…………………………………………………46
4. Neuroendocrine syndromes…………………………………………50
5. Endometriosis……………………………………………………….58
6. Anomalies of the position of the internal genital organs…………....60
7. Benign tumors…………………………………………………….…61
8. Malignant tumors…………………………………………………....64
9. Trophoblastic disease………………………………………………..67
10. Emergency conditions in gynecology……………………………….68
11. Mammary glands diseases…………………………………………...71
12. Sterility……………………………………………………………....74
13. Operative gynecology……………………………………………….76

Answers …………………………………………………………………….74
Bibliography………………………………………………………………...76

3
SECTION I. OBSTETRICS

1. Pregnancy, labor and postpartum period

1. The pregnancy is full term. The body weight of the parturient woman is 62 kg.
Fetus is in a longitudinal lie, the head is fixed to the pelvic inlet. Abdominal
circumference is 100 cm. Uterine fundus height is 35 cm. What body weight of the
fetus can be expected?
A. 3.5 kg
B. 4 kg
C. 2.5 kg
D. 3 kg
E. 4.5 kg

2. A newborn has Apgar score of 9. When should the infant be put to the breast?
A. In the delivery room
B. After 12 hours
C. After 2 hours
D. On the 2nd day
E. On the 3rd day

3. A 20-year-old woman, gravida 2, Para 1 has been in labor for 4 hours. Her
condition is satisfactory. Moderately painful contractions occur every 3 minutes
and last for 35-40 seconds.
Amniotic fluid does not spill out yet. The fetus is in longitudinal position. Fetal
heartbeats are 136/min., clear and rhythmic. Major segment of the fetal head is
fixed to the pelvic inlet. Vaginal examination shows smooth cervix of 6cm,
amniotic sac is intact, sagittal suture is in the left oblique diameter, occipital
fontanel is on the right near the symphysis pubis. What stage of the labor is it?
A. Active phase of the first stage of normal labor
B. Latent phase of the first stage of normal labor
C. The second stage of normal labor
D. Precursors of childbirth
E. Preliminary stage

4. It is the 3rd day after the normal term labor; the infant is rooming-in with the
mother and is on breastfeeding. Objectively: the mother’s general condition is
satisfactory. Temperature is 36.4°C, heart rate is 80/min., BP is 120/80 mm Hg.
Mammary glands are soft and painless; lactation is moderate, unrestricted milk
flow. The uterus is dense; the uterine fundus is located by 3 fingers below the

4
navel. Lochia are sanguino-serous, moderate in volume. Assess the dynamics of
uterine involution:
A. Physiological involution
B. Subinvolution
C. Lochiometra
D. Pathologic involution
E. Hematometra

5. A 26-year-old woman, who gave birth 7 months ago, has been suffering from
nausea, morning sickness, somnolence for the last 2 weeks. The patient breasfeeds;
no menstruation. She has been using no means of contraception. What method
would be most efficient in clarification of the diagnosis?
A. Ultrasound
B. Small pelvis radiography
C. Palpation of mammary glands and squeezing out colostrum
D. Bimanual abdominovaginal examination
E. Mirror examination

6. A parturient woman is 23 years old. Internal obstetric examination shows the


uterine cervix to be completely open. Fetal bladder is absent. Cephalic presentation
is observed in the plane of the small pelvic outlet. Sagittal suture is at the
longitudinal section of the small pelvic outlet, small fontanel is situated closer to
the uterus. What cephalic position will the newborn have during birth in this case?
A. Minor oblique lie
B. Longitudinal lie
C. Transverse lie
D. Medium oblique lie
E. Major oblique lie

7. A 23-year-old primigravida at 39 weeks gestation has been admitted to the


maternity ward with irregular contractions. The intensity of uterine contractions is
not changing, the intervals between them stay long. Bimanual examination reveals
that the cervix is centered, soft, up to 1,5 cm long. There is no cervical dilatation.
What diagnosis should be made?
A. Pregnancy I, 39 weeks, preliminary period
B. Pregnancy I, 39 weeks, labor 1,1 period, the latent phase
C. Pregnancy I, 39 weeks, labor I, period 1, the active phase
D. Pregnancy I, 39 weeks, birth 1,1 period, the acceleration phase
E. Pregnancy I, 39 weeks, pathological preliminary period

5
8. During her first visit to the prenatal clinic a pregnant woman was referred to
other doctors for mandatory consultation. The patient was refered to:
A. Internist, dentist
B. Internist, surgeon
C. Dentist, surgeon
D. Surgeon, oculist
E. Therapist, oculist

9. A 32-year-old gravida complains of episodes of unconsciousness, spontaneous


syncopes that are quickly over after a change of body position. A syncope can be
accompanied by quickly elapsing bradycardia. There are no other complications of
gestation. What is the most likely reason for such condition?
A. Postcava compresseion by the gravid uterus
B. Pressure rise in the veins of extremities
C. Pressure fall in the veins of extremities
D. Vegetative-vascular dystonia (cardiac type)
E. Psychosomatic disorders

10. A parturient woman is 25 years old, it is her second day of postpartum period.
It was her first full-term uncomplicated labour. The lochia should be:
A. Bloody
B. Sanguino-serous
C. Mucous
D. Purulent
E. Serous

11. A newborn's head is of dolichocephalic shape, that is front-to-back elongated.


Examination of the occipital region of head revealed a labour tumour located in the
middle between the small fontanel and big fontanel. The delivery took place with
the following type of fetus head presentation:
A. Posterior vertex presentation
B. Anterior vertex presentation
C. Presentation of the bregma
D. Brow presentation
E. Face presentation

12. A parturient woman is 27 year old, it was her second labour, delivery was at
term, normal course. On the 3rd day of postpartum period body temperature is
36,8oC, Ps - 72/min, AP - 120/80 mm Hg. Mammary glands are moderately
swollen, nipples are clean. Abdomen is soft and painless. Fundus of uterus is 3
fingers below the umbilicus. Lochia are bloody, moderate. What is the most

6
probable diagnosis?
A. Physiological course of postpartum period
B. Subinvolution of uterus
C. Postpartum metroendometritis
D. Remnants of placental tissue after labour
E. Lactostasis

13. Internal obstetric examination of a parturient woman revealed that the sacrum
hollow was totally occupied with fetus head, ischiadic spines couldn't be detected.
Sagittal suture is in the straight diameter, occipital fontanel is directed towards
symphysis. In what plane of small pelvis is the presenting part of the fetus?
A. Plane of pelvic outlet
B. Wide pelvic plane
C. Narrow pelvic plane
D. Plane of pelvic inlet
E. Over the pelvic inlet

14. An 18-year-old woman, at the 16th week of pregnancy, turned to a doctor for
the first prenatal examination. The exact date of the last menstruation is not known,
but, according to the patient, the pregnancy came about 5 months ago, after she
stopped taking oral contraceptives. Alpha-fetoprotein (AFP) in the blood plasma of
the mother has increased by 3 times compared with the norm.Which survey
method is most appropriate?
A. Repeated determination of AFP concentration
B. Triple screening of AFP, serum HCG, and estriol
C. Ultrasonographic study
D. Amniocentesis to measure the level of AFP concentration
E. Amniocentesis for chromosomal analysis

2. Early gestosis of pregnant women. Preeclampsia and eclampsia


1. A multigravida on the 38th week of her pregnancy complains of increased BP up
to 140/90 mm Hg, edema of the shins for 2 weeks. In the last month she gained 3.5
kg of weight. Urine analysis: protein - 0.033 g/L. Make the diagnosis:
A. Mild preeclampsia
B. Moderate preeclampsia
C. Pregnancy hypertension
D. Severe preeclampsia
E. Pregnancy edema

7
2. A woman at 30 weeks pregnant has had an attack of eclampsia at home. On
admission to the maternity ward AP- 150/100 mm Hg. Predicted fetal weight is
1500 g. There is face and shin pastosity. Urine potein is 0,66°/oo. Parturient canal
is not ready for delivery. An intensive complex therapy has been started. What is
the correct tactics of this case management?
A. Delivery by cesarean section
B. Continue therapy and prolong pregnancy for 1-2 weeks
C. Continue therapy and prolong pregnancy for 3-4 weeks
D. Labor induction by intravenous oxytocin or prostaglandins
E. Treat preeclampsia and achieve the delivery by way of conservative
management

3. A multigravida at 39 weeks of gestation has been delivered to a hospital having


a regular labour activity for 8 hours, the amniotic fluid spilled out an
hour ago. She complains of headache, seeing spots. AP is of 180/100 mm Hg.
Urine test results: protein - 3,3 g/l, hyaline cylinders. Fetal heart rate is 140/min,
rhythmical. Vaginal examination reveals complete crevical dilatation, the fetal
head is on the pelvic floor, sagittal suture is in line with obstetric conjugate, the
occipital fontanel is under the pubis. What is the optimal tactics of labour
management?
A. Outlet forceps
B. Cavity forceps
C. Cesarean section
D. Vacuum extraction of the fetus
E. Conservative labour management

4. A 28-year-old parturient complains about headache, vision impairment, psychic


inhibition. Objectively: AP- 200/110 mm Hg, evident edemata of legs and anterior
abdominal wall. Fetus head is in the area of small pelvis. Fetal heartbeats is clear,
rhythmic, 190/min. Internal examination revealed complete cervical dilatation,
fetus head was in the area of small pelvis. What tactics of labor management
should be chosen?
A. Forceps operation
B. Cesarean
C. Embryotomy
D. Conservative labor management with episiotomy
E. Stimulation of labor activity

5. A 28-years-old woman complains of nausea and vomiting about 10 times per


day. She has been found to have body weight loss and xerodermia. The pulse is

8
100 bpm. Body temperature is 37, 2oC. Diuresis is low. USI shows 5-6 weeks of
pregnancy. What is the most likely diagnosis?
A. Moderate vomiting of pregnancy
B. Mild vomiting of pregnancy
C. I degree preeclampsia
D. Premature abortion
E. Food poisoning

6. A pregnant woman in her 40th week of pregnancy have obstetric examination:


the cervix of uterus is undeveloped. The oxytocin test is negative. Examination at
32 weeks revealed: AP 140/90 mm Hg, proteinuria 1 g/l, peripheral edemata.
Reflexes are normal. Choose the most correct tactics:
A. Labour stimulation after preparation
B. Absolute bed rest for 1 month
C. Complex therapy of gestosis for 2 days
D. Caesarian section immediately
E. Complex therapy of gestosis for 7 days

7. A primagravida with pregnancy of 37-38 weeks complains of headache, nausea,


pain in epigastrium. Objective: the skin is acyanotic. Face is hydropic, there is
short fibrillar twitching of blepharons, muscles of the face and the inferior
extremities. The look is fixed. AP- 200/110 mm Hg; sphygmus of 92 bpm, intense.
Respiration rate is 32/min. Heart activity is rhythmical. Appreciable edemata of the
inferior extremities are present. Urine is cloudy. What medication should be
administered?
A. Droperidolum of 0,25% - 2,0 ml
B. Dibazolum of 1% - 6,0 ml
C. Papaverine hydrochloride of 2% - 4,0 ml
D. Hexenalum of 1% - 2,0 ml
E. Pentaminum of 5% - 4,0 ml

8. A pregnant woman for 9-10 weeks complains of vomiting to 15-20 times a day,
a significant salivation. For 2 weeks, body weight decreased by 2 kg. AT - 100/60
mm Hg. Art., pulse - 110 beats per minute. The skin is dry, pale. Pregnant is
mentally labile. Diuresis is lowered. Make a diagnosis.
A. Vomiting in pregnancy mild severity
B. Vomiting of a pregnant woman of moderate severity
C. Excessive vomiting of pregnant women
D. Botkin's disease
E. Nutritional Toxic Infection

9
9. Primigravida 23 y.o., the term of pregnancy 37-38 weeks. The condition is
difficult - at home there was an attack of eclampsia.AT - 180/100 mm Hg. Art.,
pulse - 98 beats per minute. Generalized edema. Consciousness is dizzy.Define the
tactics of the pregnant woman.
A. Urgent termination of pregnancy by cesarean section on the background of
intensive care
B. Prolongation of pregnancy in the background of intensive care
C. Intensive therapy for 2-3 days with subsequent degeneration
D. Overlays of obstetric forceps
E. Early amniotomy
10. The 22-year-old pregnant is delivered in the 38-week period in a difficult state
with a dizziness of consciousness. During the last three days there was swelling,
headache pain, nausea, one-time vomiting. Objectively: AT - 160/110 mm Hg.
Art., small fibrillary twitching of the musculature of the face, difficult nasal
breathing. During transporting, the upper limbs began to twitch, the spine curved,
the jaws tightly tightened, the breath stopped. Following this, there appeared clonic
seizures, expressed cyanosis. Then the convulsions ceased, there was a deep noisy
breath, on the lips appeared foam, painted with blood. What is the most likely
diagnosis?
A. Diabetic coma
B. Epilepsy
C. Chorea
D. Hypertensive crisis
E. Eclampsia
11. A 22-year-old primigravid woman at 35 weeks’ gestation comes to the
physician complaining of a severe frontal headache that has not improved with
acetaminophen. She also notes changes in her vision over the past 12 hours. Within
the past 6 hours, she has developed constant epigastric pain. Her temperature is
37.0 C, blood pressure is 150/90 mm Hg, pulse is 88/min, and respirations are
12/min. Examination shows moderate to severe edema in the face, hands, and feet.
Urinalysis shows 3+ proteinuria. Which of the following is the outcome of most
immediate concern in a patient with these signs and symptoms?
A. Eclampsia
B. Hepatitis
C. Migraine
D. Myocardial infarction
E. Systemic lupus erythematosus

10
3. Premature and postmature pregnancy
1. A pregnant woman is 28 years old. Anamnesis: accelerated labor complicated by
the II degree cervical rupture. The following two pregnancies resulted in
spontaneous abortions at the terms of 12 and 14 weeks. On speculum examination:
the uterine cervix is scarred from previous ruptures at 9 and 3 hours, the cervical
canal is gaping. On vaginal examination: the cervix is 2 cm long, the external os is
open 1 cm wide, the internal os is half-open; the uterus is enlarged to the 12th
week of pregnancy, soft, mobile, painless, the appendages are without changes.
What diagnosis would you make?
A. Isthmico-cervical insufficiency, habitual noncarrying of pregnancy
B. Threatened spontaneous abortion
C. Incipient abortion, habitual noncarrying of pregnancy
D. Cervical hysteromyoma, habitual noncarrying of pregnancy
E. Cervical pregnancy, 12 weeks

2. A woman, aged 40, primigravida, with infertility in the medical history, on the
42-43 week of pregnancy. Labour activity is weak. Longitudinal presentation
of the fetus, I position, anterior position. The head of the fetus is fixed to
pelvic inlet. Fetus heart rate is 140 bmp, rhythmic, muffled. Cervix dilation is 4
cm. On amnioscopy: greenish colour of amniotic fluid and fetal membranes.
Cranial bones are dense, cranial sutures and small fontanel is diminished. What
should be tactics of delivery?
A Caesarean section
B Amniotomy, labour stimulation, fetal hypoxia treatment
C Fetal hypoxia treatment, in the ІІ period - forceps delivery
D Fetal hypoxia treatment, conservative delivery
E Medication sleep, amniotomy, labour stimulation
3. A woman of 32 years old, whose second pregnancy at the 8th week of
pregnancy, first time during the pregnancy turns to a doctor. The first child was
born on 34th week due to premature birth; the pregnancy was complicated by iron
deficiency anemia. No other serious illness was observed. AT - 100/70 mm Hg.
During the review, no violations were detected. The results of ultrasonography
indicate diamiotic uterine pregnancy, which corresponds to the 8th week of
gestation.Which of the complications in the pregnant woman in the future is an
increased risk?
A. Premature placental ablation
B. Chromosomal abnormalities of the fetus
C. Hyperthyroidism

11
D. Preterm labor
E. Syndrome of fetal-fetal transfusion
4. A women's 24-year-old woman addressed a woman's consultation with
complaints of delayed menstruation for up to 10 weeks. In the history - 2
spontaneous miscarriages. During the ultrasound examination, one fetal egg was
detected, which corresponds to 6-7 weeks, fetal heartbeat is not determined.What
is the next tactic in this situation?
A. The use of uterotonic means
B. Observation for one week with subsequent ultrasonography
C. Appoint hormonal therapy
D. Сurettage of the walls of the uterus
E. Appoint antibiotic therapy
5. A 25-year-old woman has come up with complaints about remitting pain in the
lower abdomen in the 31-32 week of pregnancy. Objectively: the position of the
fetus is longitudinal, occipital presentation. Heartbeat of the fetus - 146 bp / min,
rhythmic. Internal obstetric examination: cervical opening - 5-6 cm. The fetal
bladder intact. The head of the fetus is mobile over the pelvic inlet. What is the
doctor's tactic?
A. Conducting delivery as premature
B. To carry out drug-induced tocolysis and prophylaxis of fetal distress
C. Make amniotomy
D. Inject 1 ml of 2% promedol solution for pain relief
E. Stimulation of labor activity
6. A woman 25 years old, suspected of the onset of her first pregnancy, appealed to
a woman's consultation on March 22, 2018 with complaints of lower abdominal
pain. From anamnesis - the last menstruation was January 18, 2018. In a
gynecological study: the cervix is preserved, the external os is closed, the uterine
body is increased to 9 weeks of pregnancy, mobile, painless. Set up a complication
that has arisen.
A. The risk of miscarriage in 9 weeks of pregnancy
B. Incomplete abortion
C. Leiomyoma of the uterus
D. Pregnancy has stopped
E. Molar pregnancy
7. Primigravida 38 years of age 41-42 weeks complains of weakening of the fetal
movements. Clinical and laboratory data indicate long-term pregnancy. The
estimated weight of the fetus is 4200 g. The heartbeat of the fetus is muffled, 160

12
beats per minute. According to amniocoscopy, amniotic fluid of green colon.What
is the tactic for ending pregnancy?
A. Caesarean section
B. Expect the beginning of independent maternity activities
C. Carry out an oxytocin test
D. Stimulate birth by oxytocin
E. Stimulate birth by prostaglandins

4. Anomalies of the bone pelvis.The problem of macrosomia in modern


obstetrics. Childbirth at malposition and pelvic presentations.
1. A 22-year-old woman, gravida 1, para 0 arrived with complaints of sharply
painful contractions that occur every 4-5 minutes and last for 25-30 seconds.
Amniotic fluid did not burst. The fetus is in transverse lie, fetal heartbeats are not
affected. Contraction ring is acutely painful, located obliquely at the umbilicus.
What is the most likely diagnosis?
A. Impending uterine rupture
B. Uterine tetany
C. Excessive uterine activity during labor
D. Discoordinated labor
E. Uterine rupture

2. Primigravida in 40 weeks. The position of the fetus is longitudinal, the head is


pressed to the pelvic inlet. The size of the pelvis 26-26-30-18 cm. Diagonal
conjugate is 10.5 cm. What form the pelvis??
A. Simple flat pelvis
B. Normal size of the pelvis
C. General contracted pelvis
D. Flat rakhitic pelvis
E. Transverse contracted pelvis
3. After delivery, a large fetus (4200 g) in the female D., 28 years old, appeared
pain in the area of the pubic symphysis. The intensity of pain increases
significantly after changing body position in bed and walking. Palpation is
diagnosed deepening between the pubic bones. Palpation is painful. Your tactics?
A. Fixing the pelvis with a "hammock" for 3-4 weeks
B. Тight bandaging of the pelvis for 3-4 weeks
C. One-month imposition of plaster splints
D. Wearing a bandage for two months
E. Bed mode for a month

13
4.A primipara with active maternity activities. Pregnancy first , CA - 110 cm,
HSUF - 40 cm. The size of the pelvis is 26-29-32-20 cm. The position of the fetus
is longitudinal, first position, anterior kind . In the area of the bottom of the uterus,
a large, dense part of the fetus is determined. At an internal obstetrical
examinations: the cervix is smoothed, opening 4 cm, fetal bladder intact, the fetal
leg present. What tactics childbirth?
A. Amniotomy
B. Conservative management of labor by Tsov'yanov
C. Labor induction
D. Caesarean section
E. Classical manual assistance
5. A primipara in the term of delivery. Childbirth lasts for 8 hours. Light amniotic
fluid left. Сontractions for 30-40 seconds every 5-6 minutes, regular, painful. The
head presentations, head is fixed to the pelvic inlet. Fetal heartbeat is clear, 136
bpm. At an internal obstetrical examination, opening of the uterine cervix is 8 cm,
palpable forehead, supraclavicular arches of the fetus, frontal suture in the right
oblique size. Fetal bladder intact. What kind of childbirth tactics?
A. Induction labor activity by oxytocin
B. Cavitary obstetric forceps
C. Fetus-destroying operation
D. Caesarean section
E. Sleep-rest, spasmolytics

6. Pregnant D. in the term of 38 weeks admitted to the department of the pathology


of pregnancy. Diagnosed twins: the first fetus in leg presentation, the second -in
the transverse. Determine delivery plan.
A. Urgent cesarean section
B. Childbirths through natural birth canal
C. Conduct planned caesarean section
D. Appoint corrective gymnastics
E. Take an external turn over Arkhangelsk
7. A primipara F. 24 years old was admitted to the maternity ward 16 hours after
the onset of labor. BP - 110/70 mm Hg Art., pulse - 100 beats. / min .Registered
for this pregnancy in the female consultation was not included. Objectively
determined transverse position of the fetus, heartbeat of the fetus is not bugged.In
the vagina - the hand of the fetus. Labor activity is absent. What is the diagnosis?
A. Launched transverse position of the fetus
B. Uterine rupture

14
C Septic shock
D. Hemorrhagic shock
E. Amniotic fluid embolism
8. The woman in labor in the second stage, after the birth of the buttocks of the
fetus developed a weakness of labor activity and signs of fetal distress appeared.
What is the further tactics of the doctor?
A. Provide classic manual assistance.
B. To conduct the treatment of distress
C. Start stimulation of labor
D. Continue conservative management of labor
E. Perform fetal extraction for pelvic end
9. Secundipara, labor in term, protracted. Labor activity is active. Contractions
convulsive character, sharp pain of the lower segment. Contraction ring at the
navel. The size of the pelvis - 25-28-30-17 cm. Amniotic fluid came out at the
beginning of labor. Fetal heartbeat dull, up to 100 beats. / Min. What is the most
likely diagnosis?
A. The threat of uterine rupture
B. Too strong labor activity
C. Premature detachment of the placenta
D. Discoordinated labor activity
E. Primary uterine inertia
10. A 20-year-old woman has been registered for her first pregnancy for 11-12
weeks. From anamnesis it was discovered that she grew and developed in difficult
material and living conditions. In the childhood he often suffered from infectious
diseases, he suffered rickets. At inspection found: female weight - 56 kg, height -
154 cm, blood pressure -100/60 mm Hg Art., pulse - 72 beats per minute, Hb - 100
g / l. The size of the pelvis is 26-26-31-18 cm. What factor can affect the delivery
method?
A. Age
B. Parity
C. Anemi
D. Narrow pelvis
E. Asthenic syndrome
11. A primapara with pelvis size 25-28-31-20 cm has active labor activity.
Amniotic fluid poured out, clear. Fetus weight is 4500 g, the head is fixed to the
small pelvis inlet. Vasten’s sign as positive. Cervix of uterus is fully dilated.
Amniotic sac is absent. The fetus heartbeat is clear, rhythmic, 136 bpm. What is
the labor tactics?

15
A. Stimulation of the labor activity
B. Caesarean section
C. Obstetrical forceps
D. Vacuum extraction of the fetus
E. Conservative tactics of labor

5. Abnormalities of labor activity


1. A baby was born by a young smoker. The labour was complicated by uterine
inertia, difficult delivery of the baby's head and shoulders. The baby's Apgar score
was 4. Which of the following is a risk factor for a spinal cord injury?
A. Difficult delivery of the head and shoulders
B. Young age of the mother
C. Pernicious habits
D. Uterine inertia
E. Chronic hypoxia

2. A 24-year-old primipara was hospitalised with complaints of discharge of the


amniotic waters. The uterus is tonic on palpation. The position of the fetus is
longitudinal, it is pressed with the head to pelvic outlet. Palpitation of the fetus is
rhythmical, 140 bpm, auscultated on the left below the navel. Internal examination:
cervix of the uterus is 2,5 cm long, dense, the external opening is closed, light
amniotic waters are discharged. Point out the correct component of the diagnosis:
A. Antenatal discharge of the amniotic waters
B. Early discharge of the amniotic waters
C. The beginning of the 1st stage of labour
D. The end of the 1st stage of labour
E. Pathological preterm labour

3. A 30-year-old multigravida has been in labour for 18 hours. 2 hours ago the II
stage began. Fetal heart rate is clear, rhythmic, 136/min. Vaginal examination
reveals the complete cervical dilatation, the fetal head in the pelvic outlet plane.
Sagittal suture in line with obstetric conjugate, the occipital fontanel is near the
pubis. The patient has been diagnosed with primary uterine inertia. What is the
further tactics of labour management?
A. Outlet forceps
B. Labour stimulation
C. Cesarean section
D. Skin-head Ivanov's forceps
E. Vacuum extraction of the fetus

4. A 40 week pregnant secundipara is 28 years old. Contractions are very active.

16
Retraction ring is at the level of navel, the uterus is hypertonic, in form of
hourglass. On auscultation the fetal heart sounds are dull, heart rate is 100/min. AP
of the parturient woman is 130/80 mm Hg. What is the most likely diagnosis?
A. RIisk of hysterorrhexis
B. Mazolysis
C. Disturbed labour
D. Complete hysterorrhexis
E. Attack of eclampsia

5. A maternity house has admitted a primagravida complaining of irregular, intense


labour pains that have been lasting for 36 hours. The woman is tired, failed to fall
asleep at night. The fetus is in longitudinal lie, with cephalic presentation. The
fetus heartbeat is clear and rhythmic, 145/min. Vaginal examination revealed that
the uterine cervix was up to 3 cm long, dense, with retroflexion; the external os
was closed; the discharges were of mucous nature. What is the most likely
diagnosis?
A. Pathological preliminary period
B. Uterine cervix dystocia
C. Primary uterine inertia
D. Physiological preliminary period
E. Secondary uterine inertia

6. A 20-year-old woman is having labor in time continued for 4 hours. Light


amniotic fluid came off. The fetus head is pressed to the pelvic inlet. The
anticipated fetus mass is 4000g\pm 200,0 g. Heartbeat of the fetus is normal.
Internal examination: cervix is absent, opening – 2 cm, the fetal membranes are
not present. The head is in 1-st plane of the pelvis, a sagittal suture is in the left
slanting dimension.What is the purpose of glucose-calcium-hormone - vitaminized
background conduction?
A Prophylaxes of weakness of labor activity
B Labor stimulation
C Fetus hypoxia prophylaxes
D Antenatal preparation
E Treatment of weakness of labor activity.
7. A 24 years old primipara was hospitalized with complaints about discharge of
the amniotic waters. The uterus is tonic on palpation. The position of the fetus is
longitudinal, it is pressed with the head to pelvic inlet. Palpitation of the fetus is
rhythmical, 140 bpm, auscultated on the left below the navel. Internal examination:
cervix of the uterus is 2,5 cm long, dense, the external os is closed, light amniotic
waters out of it. Point a correct component of the diagnosis:

17
A Pathological preterm labour
B Antenatal discharge of the amniotic waters
C The beginning of the 1st stage of labour
D Early discharge of the amniotic waters
E The end of the 1st stage of labour
8. A 37 y.o. primigravida woman has been having labor activity for 10 hours.
Labor pains last for 20-25 seconds every 6-7 minutes. The fetus lies in longitude,
presentation is cephalic, head is pressed to the pelvic inlet. Vaginal examination
results: cervix of uterus is up to 1 cm long, lets 2 transverse fingers in. Fetal
bladder is absent. What is the most probable diagnosis?
A Discoordinated labor activity
B Pathological preliminary period
C Secondary uterine inertia
D Normal labor activity
E Primary uterine inertia
9. A 30 y.o. parturient woman was taken to the maternity house with complaints of
having acute, regular labour pains that last 25-30 seconds every 1,5-2 minutes.
Labour activity began 6 hours ago. Uterus is in higher tonus, head of the fetus is
above the opening into the small pelvis. Fetal heartbeat is 136/min. P.V: cervical
dilatation is 4 cm, uterine fauces is spasming at a height of parodynia. Head is level
with opening into the small pelvis, it is being pushed off. What is the most
probable diagnosis?
A. Secondary uterine inertia
B. Primary uterine inertia
C. Normal labour activity
D. Discoordinated labour activity
E. Pathological preliminary period

6. Fetal hypoxia and fetoplacental insufficiency.


Immunological conflict during pregnancy
1. Examination of a Rh-negative pregnant woman at 32 weeks of gestation
revealed a four-time rise of Rh-antibody titer within 2 last weeks; the titer is 1:64.
The first two pregnancies resulted in antenatal fetal death due to hemolytic disease.
What is the optimal tactics of pregnancy management?
A. Early delivery
B. Delivery at 37 weeks of gestation
C. Screening for Rh-antibodies in 2 weeks and urgent delivery in case of
further increase of antibody titer
D. Introduction of anti-Rh (D) immunoglobulin

18
E. Ultrasound for signs of hemolytic disease of the fetus
2. A woman with blood group B(III) Rh(+) gave birth to a full-term healthy boy.
Examination on the 3rd day of the infant’s life shows him to have icteric tint to his
skin. The child has no problems with suckling, sleep is nondisturbed. The abdomen
is soft, the liver protrudes by 2 cm from under the costal margin. Complete blood
count: hemoglobin - 200 g/L, erythrocytes - 5.5 • 1012/L, total bilirubin - 62
mcmol/L, indirect bilirubin - 52 mcmol/L. What condition can be suspected?
A. Physiologic jaundice
B. Congenital hepatitis
C. Hemolytic disease of the newborn due to Rh incompatibility
D. Biliary atresia
E. Hemolytic disease of the newborn due to ABO incompatibility

3. A baby was born at 36 weeks of gestation. Delivery was normal, by natural way.
The baby has a large cephalohematoma. The results of blood count are: Hb- 120g/l,
Er- 3, 5 • 1012/l, total serum bilirubin - 123 mmol/l, direct bilirubin -11 mmol/l,
indirect - 112 mmol/l. What are the causes of hyperbilirubinemia in this case?
A. Erythrocyte hemolysis
B. Intravascular hemolysis
C. Disturbance of the conjugative function of liver
D. Bile condensing
E. Mechanical obstruction of the bile outflow

4. A 19-year-old primiparous woman with a body weight of 54,5 kg gave birth at


38 weeks gestation to a full-term live girl after a normal vaginal delivery. The girl's
weight was 2180,0 g, body length - 48 cm. It is known from history that the
woman has been a smoker for 8 years, and kept smoking during pregnancy.
Pregnancy was complicated by moderate vomiting of pregnancy from 9 to 12
weeks pregnant, edemata of pregnancy from 32 to 38 weeks. What is the most
likely cause of low birth weight?
A. Fetoplacental insufficiency
B. Low weight of the woman
C. Woman's age
D. First trimester preeclampsia
E. Third trimester preeclampsia

5. A newborn (mother's I pregnancy) weighing 3500 g presents with jaundice,


lethargy, reduced reflexes. Objectively: second grade jaundice of skin with saffron
tint, liver - +2cm, spleen - +1 cm. Urine and feces are yellow. Blood count: Hb-
100 g/l, RBCs - 3, 2 • 1012/l, WBCs -18, 7 • 109/l, mother's blood type - 0(I)

19
Rh(+), baby's blood type - A(II) Rh(-), bilirubin - 170 mmol/l, indirect fraction.
ALT, AST rates are normal. What disease is the child most likely to have?
A. Hemolytic disease of newborn, AB0-conflict
B. Perinatal hepatitis
C. Hemolytic disease of newborn, Rh-conflict
D. Biliary atresia
E. Physiologic jaundice

6. A multigravida with Rh-isosensitization was found to have a decrease in anti-Rh


titer from 1:32 to 1:8 at 33-34 weeks of gestation. Ultrasound revealed double
contour of head, ebnlargement of fetal liver, placental thickness of 50 mm. The
patient has indication for:
A. Premature delivery
B. Course of desensitizing therapy
C. Plasmapheresis
D. Repeated (after 2 weeks) USI
E. Administration of anti-Rh gamma globulin

7. At term of a gestation of 40 weeks height of standing of a uterine fundus is less


then assumed for the given term. The woman has given birth to the child in
weight of 2500 g, a length of a body 53 cm, with an assessment on a scale of
Apgar of 4-6 points. Labor were fast. The cause of such state of the child were:
A Chronic fetoplacental insufficiency
B Delay of an intra-uterine fetation
C Placental detachment
D Infection of a fetus
E Prematurity
8. A 28 year old woman had the second labour and born a girl with manifestations
of anemia and progressing jaundice. The child’s weight was 3 400 g, the length
was 52 cm. The woman’s blood group is B (III) Rh-, the fathers blood group is A
(III) Rh+, the child’s blood group is B (III) Rh+. What is the cause of anemia?
A Antigen A incompatibility
B Antigen AB incompatibility
C Intrauterine infection
D Rhesus incompatibility
E Antigen B incompatibility

7. Bleeding in early pregnancy


1. A primigravida at the term of 20 weeks complains of pain in her lower abdomen,

20
smearing blood-streaked discharge from the genital tracts. Uterine tone is
increased, fetus is mobile. On vaginal examination: the uterus is enlarged
according to the term, uterine cervix is shortened up to 0,5 cm, external cervical os
is open by 2 cm. What is the most likely diagnosis?
A. Risk of late abortion with hemorrhage
B. Risk of late abortion without hemorrhage
C. The process of late abortion
D. Incomplete late abortion
E. Attempted late abortion

2. A 10 week pregnant woman was admitted to a hospital for recurrent pain in the
lower abdomen, bloody discharges from the genital tracts. The problems developed
after a case of URTI. The woman was registered for antenatal care. Speculum
examination revealed cyanosis of vaginal mucosa, clean cervix, open cervical
canal discharging blood and blood clots; the lower pole of the gestational sac was
visible. What tactics should be chosen?
A. Curettage of the uterus
B. Pregnancy maintenance therapy
C. Expectant management, surveillance
D. Hysterectomy
E. Antiviral therapy

3. An ambulance delivered a 21-year-old woman to the gynaecological department


with complaints of colicky abdominal pain and bloody discharges from the genital
tracts. Bimanual examination revealed that uterus was soft, enlarged to the size of
6 weeks of gestation, a gestational sac was palpated in the cervical canal. Uterine
appendages weren't palpable. Fornices are free, deep and painless. Discharges from
the genital tracts are bloody and profuse. What is the most likely diagnosis?
A. Abortion in progress
B. Cervical pregnancy
C. Threat of abortion
D. Incipient abortion
E. Interrupted fallopian pregnancy

4. A primagravida in her 20th week of gestation complains about pain in her lower
abdomen, blood smears from the genital tracts. The uterus has an increased tonus,
the patient feels the fetus movements. Bimanual examination revealed that the
uterus size corresponded the term of gestation, the uterine cervix was dilated to 0,5
cm, the external os was open by 2 cm. The discharges were bloody and smeary.
What is the most likely diagnosis?
A. Incipient abortion

21
B. Risk of abortion
C. Abortion in progress
D. Incomplete abortion
E. Missed miscarriage

5. A 26 y.o. woman complains of a mild bloody discharge from the vagina and
pain in the lower abdomen. She has had the last menstruation 3,5 months ago. The
pulse is 80 bpm. The blood pressure (BP) is 110/60 mm Hg and body temperature
is 36,6°C. The abdomen is tender in the lower parts. The uterus is enlarged up to
12 weeks of gestation. What is your diagnosis?
A Dysfunctional bleeding
B Inevitable abortion (Imminents abortion )
C Incomplete abortion
D Incipient abortion
E Complete abortion
6. A 24-year-old woman comes to the physician 2 weeks after experiencing a
spontaneous abortion at 6 weeks' gestation. She has no vaginal bleeding,
abdominal pain, fevers, or chills. Examination is unremarkable, including a normal
pelvic examination. She states that this was her first pregnancy and she wants to
know whether she and her husband need testing to determine why the miscarriage
occurred. After comforting the patient, which of the following is the most
appropriate response?
A. Investigation is initiated after the first, first- trimester miscarriage
B. Investigation is initiated after two consecutive first-trimester miscarriages
C. Investigation is initiated after three consecutive first-trimester miscarriages
D. Investigation is initiated after four consecutive first-trimester miscarriages
E. There is no need to investigate recurrent miscarriages

8. Obstetrical bleeding. Obstetric injuries.


1. A 30-year-old parturient woman was delivered to a maternity hospital with full-
term pregnancy. She complains of severe pain in the uterus that started 1 hour ago,
nausea, vomiting, cold sweat. Anamnesis states cesarean section 2 years ago.
Uterine contractions stopped. Skin and mucous membranes are pale. Heart rate is
100/min., BP is 90/60 mm Hg. Uterus has no clear margins, is sharply painful. No
heartbeat can be auscultated in the fetus. Moderate bloody discharge from the
uterus can be observed. Uterus cervix is 4 cm open. Presenting part is not visible.
The most likely diagnosis is:
A. Uterine rupture
B. Initial uterine rupture

22
C. Threatened uterine rupture
D. Premature detachment of normally positioned placenta
E. Compression of inferior pudendal vein

2. A woman is 34 years old, it is her tenth labor at full term. It is known from the
anamnesis that the labor started 11 hours ago, labor was active, painful
contractions started after discharge of waters and became continuous. Suddenly the
parturient got knife-like pain in the lower abdomen and labor activity stopped.
Examination revealed positive symptoms of peritoneum irritation, ill-defined
uterus outlines. Fetus was easily palpable, mobile. Fetal heartbeats wasn't
auscultable. What is the most probable diagnosis?
A. Rupture of uterus
B. Uterine inertia
C. Discoordinated labor activity
D. Risk of uterus rupture
E. II labor period
3. Examination of placenta revealed a defect. An obstetrician performed manual
investigation of uterine cavity, uterine massage. Prophylaxis of endometritis in the
postpartum period should involve the following actions:
A. Antibacterial therapy
B. Instrumental revision of uterine cavity
C. Haemostatic therapy
D. Contracting agents
E. Intrauterine instillation of dioxine

4. A puerperant is 32 years old, it's her first childbirth, term precipitate labor, the
III period is unremarkable, the uterus is contracted, tight. Examination of the birth
canal revealed a rupture in the left posterior vaginal wall that was closed with
catgut. Two hours later, the patient complained of a feeling of pressure on the anus,
pain in the perineum, minor vaginal discharges, edema of the vulva. These clinical
presentations are indicative most likely of:
A. Vaginal hematoma
B. Hysterocervicorrhexis
C. Hemorrhoids
D. Hysterorrhesis
E. Hypotonic bleeding

5. A 30-year-old gravida consulted a gynecologist about bright red bloody


discharges from the vagina in the 32 week of gestation. She was hospitalized with
a suspicion of placental presentation. Under what conditions is it rational to
conduct the internal examination in order to make a diagnosis?

23
A. In the operating room prepared for the operation
B. In the examination room of antenatal clinic
C. In the admission ward of maternity hospital
D. In the delivery room keeping to all the aseptics regulations
E. The examination is not to be conducted because of risk of profuse
hemorrhage
6. 10 minutes after delivery a woman discharged placenta with a tissue defect 5x6
cm large. Discharges from the genital tracts were profuse and bloody. Uterus tonus
was low, fundus of uterus was located below the navel. Examination of genital
tracts revealed that the uterine cervix, vaginal walls, perineum were intact. There
was uterine bleeding with following blood coagulation. Your actions to stop the
bleeding:
A. To make manual examination of uterine cavity
B. To apply hemostatic forceps upon the uterine cervix
C. To introduce an ether-soaked tampon into the posterior fornix
D. To put an ice pack on the lower abdomen
E. To administer uterotonics

7. A 28-year-old patient underwent endometrectomy as a result of incomplete


abortion. Blood loss was at the rate of 900 ml. It was necessary to start
hemotransfusion. After transfusion of 60 ml of erythrocytic mass the patient
presented with lumbar pain and fever which resulted in hemotransfusion stoppage.
20 minutes later the patient's condition got worse: she developed adynamia,
apparent skin pallor, acrocyanosis, t- 38,5oC, Ps-110/min, AP- 70/40 mm Hg.
What is the most likely diagnosis?
A. Hemotransfusion shock
B. Hemorrhagic shock
C. Septic shock
D. Anaphylactic shock
E. DIC syndrome

8. A 26 year old woman had the second labour within the last 2 years with
oxytocin application. The child's weight is 4080 g. After the placent birth there
were massive bleeding, signs of hemorrhagic shock. Despite the injection of
contractive agents, good contraction of the uterus and absence of any cervical and
vaginal disorders, the bleeding proceeds. Choose the most probable cause of
bleeding:
A. Atony of the uterus
B. Injury of cervix of the uterus
C. Hysterorrhexis

24
D. Delay of the part of placenta
E. Hypotonia of the uterus

9. Immediately after delivery a woman had haemorrhage, blood loss exceeded


postpartum haemorrhage rate and was progressing. There were no symptoms of
placenta detachment. What tactics should be chosen?
A. Manual removal of placenta and afterbirth
B. Uterus tamponade
C. Instrumental revision of uterine cavity walls
D. Removal of afterbirth by Crede's method
E. Intravenous injection of methylergometrine with glucose

10. A woman was hospitalised with full-term pregnancy. Examination: the uterus
is tender, the abdomen is tense, cardiac tones of the fetus are not auscultated. What
is the most probable complication of pregnancy?
A Premature detachment of normally posed placenta
B Premature labor
C Back occipital presentation
D Acute hypoxia of a fetus
E Hydramnion
11. A 34 y.o. woman in her 29-th week of pregnancy, that is her 4-th labor to
come, was admitted to the obstetric department with complaints of sudden and
painful bloody discharges from vagina that appeared 2 hours ago. The discharges
are profuse and contain grumes. Cardiacfunnction of the fetus is rhytmic, 150
strokes in the minute, uterus tone is normal. The most probable provisional
diagnosis will be:
A Placental presentation
B Detachment of normally located placenta
C Vasa previa
D Bloody discharges
E Disseminated intravascular coagulation syndrome
12. A pregnant woman (35 weeks), aged 25, was admitted to the hospital because
of bloody discharges. In her medical history there were two artificial abortions. In
a period of 28-32 weeks there was noted the onset of hemorrhage and USD showed
a placental presentation. The uterus is in normotonus, the fetus position is
transversal (Ist position). The heartbeats is clear, rhythmical, 140 bpm. What is the
further tactics of the pregnant woman care?
A To perform a delivery by means of Cesarean section
B To perform the hemotransfusion and to prolong the pregnancy

25
C To introduct the drugs to increase the blood coagulation and continue
observation
D Stimulate the delivery by intravenous introduction of oxytocin
E To keep the intensity of hemorrhage under observation and after the
bleeding is controlled to prolong the pregnancy
13. Condition of a parturient woman has been good for 30 minutes after live birth:
uterus is thick, globe-shaped; its bottom is at the level of umbilicus, bleeding is
absent. The clamp put on the umbilical cord remains at the same level, when the
woman takes a deep breath or she is being pressed over the symphysis with the
verge of hand, the umbilical cord draws into the vagina. Bloody discharges from
the sexual tracts are absent. What is the doctor’s further tactics?
A To do manual removal of afterbirth
B To apply Credes method
C To introduct oxitocine intravenously
D To do curettage of uterine cavity
E To apply Abduladze method
14. The woman who has delivered twins has early postnatal hypotonic uterine
bleeding reached 2,5% of her bodyweight. The bleeding is going on. Conservative
methods to arrest the bleeding have been found ineffective. The conditions of
patient are pale skin, acrocyanosis, oliguria. The woman is confused. The pulse is
130 bpm, BP– 75/50 mm Hg. What is the further treatment?
A Uterine extirpation
B Uterine vessels ligation
C Putting clamps on the uterine cervix
D Inner glomal artery ligation
E Supravaginal uterine amputation
15. A 20 y.o. pregnant woman with 36 weeks of gestation was admitted to the
obstetrical hospital with complains of pain in the lower abdomen and bloody
vaginal discharge. The general condition of the patient is good. Her blood pressure
is 120/80 mm Hg. The heart rate of the fetus is 140 bpm, rhythmic. Vaginal
examination: the cervix of the uterus is formed and closed. The discharge from
vagina is bloody up to 200 ml per day. The head of the fetus is located high above
the pelvic inlet. A soft formation was defined through the anterior fornix of the
vagina. What is the probable diagnosis?
A Threatened premature labor
B Incipient abortion
C Premature placental separation
D Uterine rupture

26
E Placental presentation
16. A 34-year-old woman, gravida 4, para 3 at 38 weeks’ gestation, comes to the
labor and delivery ward because of contractions. Her prenatal course was
significant for low maternal weight gain. She had a normal 18-week ultrasound
survey of the fetus and normal 36-week ultrasound to check fetal presentation. Her
blood type is O positive, and she is rubella immune. Three years ago, she had a
multiple myomectomy. She takes prenatal vitamins and has no known drug
allergies. She smokes one pack of cigarettes per day. Which of the following
complications is most likely to occur?
A. Amniotic fluid embolism
B. Anencephaly
C. Macrosomia
D. Rh isoimmunization
E. Uterine rupture

9. Extragenital diseases and pregnancy


1. A woman is on the 32nd week of her second pregnancy. She complains of fever,
chills, nausea, vomiting, lumbar pain, and dysuria. Costovertebral angle tenderness
is present on both sides. Urine analysis: pyuria, bacteriuria. Blood test:
leukocytosis. What is the most likely diagnosis?
A. Gestational pyelonephritis
B. Cystitis
C. Pyelitis
D. Glomerulonephritis
E. Latent bacteriuria

2. An 18-year-old primigravida in her 27-28 week of gestation underwent an


operation on account of acute phlegmonous appendicitis. In the postoperative
period it is necessary to take measures for prevention of the following pegnancy
complication:
A. Noncarrying of pregnancy
B. Intestinal obstruction
C. Fetus hypotrophy
D. Premature placenta detachment
E. Late gestosis

3. A woman of a high-risk group (chronic pyelonephritis in anamnesis) had vaginal


delivery. The day after labour she complained of fever and loin pains, frequent
urodynia. Specify the most probable complication:

27
A. Infectious contamination of the urinary system
B. Thrombophlebitis of veins of the pelvis
C. Infectious hematoma
D. Endometritis
E. Apostasis of sutures after episiotomy

4. A 34 year old woman in the 10th week of gestation (the second pregnancy)
consulted a doctor of antenatal clinic in order to be registered there. In the
previous pregnancy hydramnion was observed, the child's birth weight was 4086
g. What examination method should be applied in the first place?
A. The test for tolerance to glucose
B. Determination of the contents of fetoproteinum
C. Bacteriological examination of discharges from vagina
D. A cardiophonography of fetus
E. US of fetus

5. A pregnant woman may be diagnosed with hepatitis if it is confirmed by the


presence of elevated:
A SGOT (ALT)
B Sedimentation rates
C WBCs
D Alkaline phosphatase
E BUN
6. A 26-year-old nulligravid patient presents to her physician seeking
preconceptional advice. She plans to conceive in about 1 year. Her past medical
history is significant for chickenpox as a child. She had an appendectomy 2 years
ago. She takes no medications and is allergic to penicillin. Her complete physical
examination, including a pelvic examination, is unremarkable. Which of the
following is the most appropriate next step in diagnosis to prevent morbidity in this
patient’s offspring?
A. Blood cultures
B. Group B Streptococcus culture
C. Pelvic ultrasound
D. Rubella titer
E. Urine culture

7. A 26-year-old black gravida 2, para 1, at 32 weeks' gestation presents to the


physician for a prenatal visit. Her prenatal course has been remarkable for hyper
emesis gravidarum in the first trimester. She also had a urine culture in the first
trimester that grew out Group B Streptococcus. She has had type 1 diabetes for the

28
past 2 years and has had good control of her blood glucose levels during this
pregnancy. Her first pregnancy resulted in a low transverse cesarean section for
dystocia. Other than insulin, she takes no medicines and has no known drug
allergies. After a routine prenatal visit, the physician sends her to the antepartum
fetal testing unit to undergo a nonstress test (NST). Which of the following
characteristics makes this patient a good candidate for antepartum fetal testing with
an NST?
A. Black race
B. Diabetes mellitus
C. Group B Streptococcus urine culture
D. History of cesarean section
E. Hyperemesis gravidarum

8. A 19-year-old gravida 2, para 1 woman presents at her first prenatal visit


complaining of a rash, hair loss, and spots on her tongue. Her temperature is 37.0
C, blood pressure is 112/74 mm Hg, pulse is 68/min, and respirations are 14/min.
Physical examination is significant for a maculopapular rash on her trunk and
extremities, including her palms and soles. She has “moth-eaten” alopecia and
white patches on her tongue. Her uterus is 10-week size, which is consistent with
her dating by last menstrual period. The rest of her examination is unremarkable.
RPR and MHA-TP are positive. Which of the following is the most appropriate
pharmacotherapy?
A. Clindamycin
B. Gentamicin
C. Nitrofurantoin
D. Penicillin
E. Tetracycline

9. A 34-year-old woman, gravida 3, para 2, at 16 weeks' gestation comes to the


physician concerned that she may have been exposed to an infectious disease.
Yesterday, she and her 5-year-old son spent a day at the beach with one of his
classmates. This morning, the classmate was sent home from school with a fever
and rash that the teacher thought were suspicious for chickenpox. The patient is
unsure whether she had chickenpox as a child. Her temperature is 37 C, blood
pressure is 100/70 mm Hg, pulse is 88/min, and respirations are 16/min. Her
examination is unremarkable. An inquiry made by the physician confirms that the
classmate has chickenpox. Which of the following is the most appropriate next step
in management?
A. Check an IgG varicella serology
B. Wait to see whether a rash develops

29
C. Administer IV acyclovir
D. Administer oral acyclovir
E. Administer varicella vaccine
10. A 26-year-old primigravid woman at 10-weeks’ gestation comes to the
physician for a routine prenatal appointment. Her dating is based on a 6-week
ultrasound. She has sickle-cell anemia. She has no past surgical history, takes
prenatal vitamins, and has no known drug allergies. She tells the physician that she
recently learned that the father of the baby has sickle-cell trait. On examination,
her uterus is appropriate for a 10-week gestation, and fetal heart tones are heard.
Her hematocrit is 37%. What is the most appropriate next step in the management
of this patient?
A. Genetic counseling
B. Obstetric ultrasound
C. Hydroxyurea
D. IV hydration
E. Blood transfusion

11. A 17-year-old woman, gravida 1, para 0, at 38 weeks’ gestation comes to the


labor and delivery ward because of contractions. Her dating was determined by a
7- week ultrasound. Her prenatal course was complicated by gestational diabetes.
Her past surgical history is significant for shoulder surgery. She takes insulin and
prenatal vitamins. She has no known drug allergies. She smokes 3 to 4 cigarettes
per day. She is initially found to be 4 cm dilated and is contracting every 2 to 3
minutes. She is admitted to the labor and delivery ward and, over the next 4 hours,
progresses to full dilation. After pushing for 2 hours, she delivers the fetal head but
has great difficulty delivering the fetal shoulders. Eventually, the fetus is delivered
by the posterior arm. In the process of delivery the newborn’s humerus is fractured.
Which of the following factors contributed the most to the difficult delivery of the
fetus?
A. Cigarette smoking
B. Gestational age
C. Gestational diabetes
D. Maternal age
E. Maternal shoulder surgery

12. A 23-year-old woman, gravida 2, para 1 at 26 weeks’ gestation, comes to the


physician because of fevers and pain in the middle of the back on the right side.
Her fevers started 2 days ago, and the back pain began yesterday. Her temperature
is 38.3 C, blood pressure is 110/70 mm Hg, pulse is 110/min, and respirations are

30
16/min. She has left costovertebral angle tenderness. Her abdomen is benign and
gravid. Her laboratory values show leukocytes of 18,000/mm3. Urinalysis reveals
white blood cells that are too numerous to count per high- powered field. Which of
the following is the most appropriate pharmacotherapy for this patient?
A. Acyclovir
B. Cefazolin
C. Levofloxacin
D. Metronidazole
E. Tetracycline

10.Infections during pregnancy. Postpartum infectious complications

1. A woman complains of temperature increase up to 39°C, sharp pains in her


lower abdomen, and sanguinopurulent discharge from her genital tracts. From her
case history it is known that 6 days ago she underwent illegal abortion. Objectively
her blood pressure is 100/60 mm Hg, pulse is 110/min. Abdominal rigidity,
rebound tenderness (Bloomberg’s sign), and painful palpation of the lower
abdomen are observed. On bimanual examination the uterus is enlarged up to 7
weeks of pregnancy, painful, and soft; posterior vaginal fornix overhangs. Make
the diagnosis:
A. Pelviperitonitis
B. Endometritis
C. Acute adnexitis
D. Pyosalpinx
E. Metroendometritis

2. On the 9th day after childbirth the obstetric patient developed high fever up to
38° C. She complains of pain in the right mammary gland. The examination
revealed the following: a sharply painful infiltrate can be palpated in the right
mammary gland, the skin over the infiltrate is red, subareolar area and nipple are
swollen and painful. What is your diagnosis?
A. Abscess of the right mammary gland
B. Mastopathy
C. Cancer of the right mammary gland
D. Serous mastitis
E. Fibrous cystic degeneration of the right mammary gland

3. A postparturient woman, who has been breastfeeding for 3 weeks, made an


appointment with the doctor. For the last 6 days she has been feeling unwell,
complains of body temperature of 38-39°C, general weakness; within the last 2

31
days she developed pain and redness in the area of her right mammary gland.
Examination revealed her mammary gland to be significantly enlarged and
deformed; breast tissue fluctuations and lymphadenitis are observed. What type of
mastitis is the most likely?
A. Phlegmonous mastitis
B. Serous mastitis
C. Infiltrative mastitis
D. Lactostasis
E. Mammary edema

4. A parturient woman complains of pain in her mammary gland. In the painful


area there is an infiltration 3x4 cm in size with softened center. Body temperature
is 38,5oC. What is the most likely diagnosis?
A. Acute suppurative mastitis
B. Pneumonia
C. Pleurisy
D. Milk retention
E. Birth trauma

5. A 20-year-old woman on the 10th day after her discharge from the maternity
ward developed fever up to 39oC and pain in her left mammary gland. On
examination the mammary gland is enlarged, in its upper outer quadrant there is a
hyperemic area. In this area a dense spot with blurred margins can be palpated. The
patient presents with lactostasis and no fluctuation. Lymph nodes in the right
axillary crease are enlarged and painful. Specify the correct diagnosis:
A. Lactational mastitis
B. Abscess
C. Erysipelas
D. Dermatitis
E. Tumor

6. A woman addressed a gynecologist on the 20th day of puerperal period with


complaints of pain in the left mammary gland, purulent discharge from the nipple.
Objectively: Ps- 120/min., body temperature is 39oC. The left mammary gland is
painful, larger than the right one, the skin there is hyperemic; in the upper quadrant
there is an infiltrate 10x15 cm in size with soft center. Blood test: ESR-50
mm/hour, leukocytes - 15,0 • 109/l. What would be the treatment tactics?
A. Transfer to a surgical department for surgical treatment
B. Refer to a gynecology department
C. Refer to a postnatal department
D. Refer to a surgeon for conservative treatment

32
E. Lance the mammary gland abscess in a maternity department
7. 2 weeks after labour a parturient woman developed breast pain being observed
for 3 days. Examination revealed body temperature at the rate of 39°C, chills,
weakness, hyperaemia, enlargement, pain and deformity of the mammary gland.
On palpation the infiltrate was found to have an area of softening and fluctuation.
What is the most likely diagnosis?
A. Infiltrative-purulent mastitis
B. Phlegmonous mastitis
C. Lactostasis
D. Serous mastitis
E. Mastopathy

8. A maternity patient breastfeeding for 1,5 weeks has attended a doctor. She
considers the onset of her disease to be when proportional breast engorgement
occurred. Mammary glands are painful. Body temperature is 36,6°C. Expression of
breast milk is hindered. The most likely diagnosis is:
A. Lactostasis
B. Infiltrative mastitis
C. Suppurative mastitis
D. Chronic cystic mastitis
E. Gangrenous mastitis

9. On the 10th day postpartum a puerperant woman complains of pain and


heaviness in the left mammary gland. Body temperature is 38,8°C, Ps- 94 bpm.
The left mammary gland is edematic, the supero-external quadrant of skin is
hyperemic. Fluctuation symptom is absent. The nipples discharge drops of milk
when pressed. What is a doctor's further tactics?
A. Antibiotic therapy, immobilization and expression of breast milk
B. Compress to both mammary glands
C. Inhibition of lactation
D. Physiotherapy
E. Opening of the abscess and drainage of the mammary gland

10. A 32-year-old pregnant woman at the term of 5-6 weeks was vaccinated
against influenza along with her whole family. At that time she was not aware of
her pregnancy. The pregnancy is wanted. The woman needs an advice from the
family doctor regarding the maintenance of her pregnancy, namely whether there is
a risk of fetal malformations because of received vaccination. What advice should
the doctor give in this case?
A. Vaccination against influenza is safe during pregnancy

33
B. Therapeutic abortion is recommended
C. Immediate ultrasound of the lesser pelvis is necessary
D. Test for antibodies against influenza virus is necessary
E. An infectious diseases specialist must be consulted

11. On the 5th day after labor body temperature of a 24-year-old parturient
suddenly rose up to 38, 7oC. She complains about weakness, headache, abdominal
pain, irritability. Objectively: AP- 120/70 mm Hg, Ps- 92 bpm, to- 38, 7oC.
Bimanual examination revealed that the uterus was enlarged up to 12 weeks of
pregnancy, it was dense, slightly painful on palpation. Cervical canal lets in 2
transverse fingers, discharges are moderate, turbid, with foul smell. In blood:
skeocytosis, lymphopenia, ESR - 30 mm/h. What is the most likely diagnosis?
A. Endometritis
B. Parametritis
C. Pelviperitonitis
D. Metrophlebitis
E. Lochiometra

12. On the first day after labour a woman had the rise of temperature up to 39°C.
Rupture of fetal membranes took place 36 hours before labour. Examination of the
bacterial flora of cervix of the uterus revealed hemocatheretic streptococcus of A
group. The uterus body is soft, tender. Discharges are bloody, with admixtures of
pus. Specify the most probable postnatal complication:
A. Metroendometritis
B. Thrombophlebitis of veins of the pelvis
C. Infectious hematoma
D. Infective contamination of the urinary system
E. Apostasis of sutures after the episiotomy

11. Operative obstetrics


1. 6 hours ago the waters of a 30-year-old gravida 1, para 0, burst; her preliminary
period was pathologic and lasted for over 2 days; the term of pregnancy is 39
weeks. No labor activity is observed. Fetal head presents above the pelvic inlet.
Fetal heartbeats are 142/min., clear and rhytmic. On vaginal examination the
uterine cervix is not dilated. What further tactics should the doctor choose?
A. Perform cesarean section
B. Induce cervical dilation with prostaglandins
C. Stimulate the labor with oxytocin
D. Wait for the onset of spontaneous labor

34
E. Prolong the pregnancy, while providing antibacterial treatment
2. A 24-year-old pregnant woman on her 37th week of pregnancy has been
delivered to a maternity obstetric service with complaints of weak fetal
movements. Fetal heart beats are 95/min. On vaginal examination the uterine
cervix is tilted backwards, 2 cm long, external orifice allows inserting a
fingertip. Biophysical profile of the fetus equals 4 points. What tactics of
pregnancy management should be chosen?
A. Urgent delivery via cesarean section
B. Treatment of placental dysfunction and repeated analysis of the fetal
biophysical profile on the next day
C. Doppler measurement of blood velocity in the umbilical artery
D. Urgent preparation of the uterine cervix for delivery
E. Treatment of fetal distress, if ineffective, then elective cesarean section on
the next day
3. A 30-year-old multigravida has been in labour for 18 hours. 2 hours ago the
pushing stage began. Fetal heart rate is clear, rhythmic, 136/min. Vaginal
examination reveals complete cervical dilatation, the fetal head in the pelvic outlet
plane. Sagittal suture is in line with obstetric conjugate, the occipital fontanel is
near the pubis. The patient has been diagnosed with primary uterine inertia. What
is the further tactics of labour management?
A. Outlet forceps
B. Labour stimulation
C. Cesarean section
D. Skin-head Ivanov's forceps
E. Vacuum extraction of the fetus
4. A multigravida at 39 weeks of gestation presenting with regular labour activity
for 8 hours has been delivered to a hospital; the waters broke an hour ago. She
complains of headache, seeing spots. BP is 180/100 mm Hg. Urine test results:
protein - 3,3 g/l, hyaline cylinders. Fetal heart rate is 140/min, rhythmical. Vaginal
examination reveals complete cervical dilatation, the fetal head is on the pelvic
floor, sagittal suture is in line with obstetric conjugate, the occipital fontanel is
under the pubis. What is the optimal tactics of labour management?
A. Outlet forceps
B. Cavity forceps
C. Cesarean section
D. Vacuum extraction of the fetus
E. Conservative labour management

35
5. During the dynamic observation of a parturient woman in the second stage of
labor it was registered that the fetal heart rate decreased to 90-100/min. and did not
normalize after contractions. Vaginal examination revealed the complete cervical
dilatation, the fetal head filling the entire posterior surface of the pubic symphysis
and sacral hollow; the sagittal suture was in the anteroposterior diameter of the
pelvic outlet, the posterior fontanelle was in front under the pubic arch. What plan
for further labour management should be recommended?
A. Application of forceps minor
B. Caesarean section
C. Episiotomy
D. Application of cavity forceps
E. Stimulation of labour activity through intravenous injection of oxytocin

6. A secundipara has regular birth activity. Three years ago she had cesarean
section for the reason of acute intrauterine hypoxia. During parodynia she
complains of extended pain in the area of postsurgical scar. Objectively: fetus
pulse is rhythmic - 140 bpm. Vaginal examination shows 5 cm cervical dilatation.
Fetal bladder is intact. What is the tactics of choice?
A. Cesarean section
B. Augmentation of labour
C. Obstetrical forceps
D. Waiting tactics of labor management
E. Vaginal delivery

7. A 22-year-old primigravid woman at 32 weeks’ gestation comes to the


emergency department because of heavy vaginal bleeding and abdominal pain. Her
prenatal course was unremarkable, including a normal 20- week ultrasound.
Physical examination demonstrates a contracted uterus with hypertonus. A large
“gush” of blood occurs during the cervical examination, which demonstrates a long
and closed cervix. The fetal heart rate tracing shows severe late decelerations.
Which of the following is the most appropriate next step in management?
A. Expectant management
B. Magnesium sulfate
C. Oxytocin
D. Terbutaline
E. Cesarean section
8. A woman aged 40, primigravida, with infertility in the medical history, on the
42-43 week of pregnancy. Labour activity is weak. Longitudinal presentation of
the fetus, I position, anterior position. The head of the fetus is engaged to pelvic
inlet. Fetus heart rate is 140 bmp, rhythmic, muffled. Cervix dilation is 4 cm. On

36
amnioscopy: greenish color of amniotic fluid and fetal membranes. Cranial bones
are dense, cranial sutures and small fontanels are diminished. What should be
tactics of delivery?
A. Caesarean section
B. Fetal hypoxia treatment, in the ІІ period - forceps delivery
C. Medication sleep, amniotomy, labour stimulation
D. Fetal hypoxia treatment, conservative delivery
E. Amniotomy, labour stimulation, fetal hypoxia treatment
9. A 28-year-old parturient complains about headache, vision impairment, psychic
inhibition. Objectively: AP- 200/110 mm Hg, evident edemata of legs and anterior
abdominal wall. Fetus head is in the area of small pelvis. Fetal heartbeats is clear,
rhythmic, 190/min. Internal examination revealed complete cervical dilatation,
fetus head was in the area of small pelvis. What tactics of labor management
should be chosen?
A. Inlet Forceps operation
B. Outlet Forceps operation
C. Embryotomy
D. Stimulation of labor activity
E. Conservative labor management with episiotomy
10. A secondipara has regular birth activity. Three years ago she had cesarean
section for the reason of acute intrauterine hypoxia. During parodynia she
complains of extended pain in the area of postsurgical scar. Objectively: fetus
pulse is rhythmic - 140 bpm. Vaginal examination shows 5 cm cervical dilatation.
Fetal bladder is intact. What is the tactics of choice?
A. Waiting tactics of labor management
B. Vaginal delivery
C. Augmentation of labour
D. Obstetrical forceps
E. Cesarean section
11. A 30 y.o. primigravida woman has got intensive labor pain every 1-2 minutes
that lasts 50 seconds. The disengagement has started. The perineum with the height
of 4 cm has grown pale. What actions are necessary in this situation?
A. Vacuum extraction of fetus
B. Expectant management
C. Perineum protection
D. Perineotomy
E. Episiotomy

37
12. A 30 y.o. woman has the 2-nd labour that has been lasting for 14 hours.
Heartbeat of fetus is muffled, arrhythmic, 100/min. Vaginal examination: cervix of
uterus is completely opened, fetus head is level with outlet from small pelvis.
Saggital suture is in the straight diameter, small crown is near symphysis. What is
the further tactics of handling the delivery?
A. Use of obstetrical forceps
B. Cesarean section
C. Use of cavity forceps
D. Cranio-cutaneous (Ivanovs) forceps
E. Stimulation of labour activity by oxytocin
13. A woman at 30 weeks pregnant has had an attack of eclampsia at home. On
admission to the maternity ward AP is 180/100 mm Hg. Predicted fetal weight is
1500 g. There is face and shin pastosity. Urine protein is 3,6 g/d. Parturient canal is
not ready for delivery. An intensive complex therapy has been started. What is the
correct tactics of this case management?
A. Continue therapy and prolong pregnancy for 3-4 weeks
B. Continue therapy and prolong pregnancy for 1-2 weeks
C. Delivery by cesarean section
D. Labor induction by intravenous oxytocin or prostaglandins
E. Treat preeclampsia and achieve the delivery by way of conservative
management

38
Section II. GYNECOLOGY
1. Methods of examination of gynecologic patients
1. During regular preventive gynecological examination a 30-year-old woman
was detected to have dark blue punctulated ’’perforations” on the vaginal
portion of the uterine cervix.The doctor suspects endometriosis of the vaginal
portion of the uterine cervix. What investigation method would be most
informative for diagnosis confirmation?
A. Colposcopy, target biopsy of the cervix
B. US of the lesser pelvis
C. Hysteroscopy
D. Curettage of the uterine cavity
E. Hormone testing

2. A 26-year-old woman came to a gynecologist for a regular check-up. She has no


complaints. Per vaginum: the uterus lies in anteflexion, not enlarged, dense,
mobile, painless. On the left from the uterus in the area of uterine appendages there
is a mobile painless outgrowth that can be moved independently from the uterus.
On the right the appendages cannot be detected. What additional investigation
would be informative for diagnosis clarification?
A. Ultrasound of the lesser pelvis
B. Metrosalpingography
C. Examination for urogenital infection
D. Colposcopy
E. Colonoscopy

3. A 59-year-old female patient attended a maternity welfare clinic with complains


of bloody discharge from the genital tracts. Postmenopause is 12 years. Vaginal
examination revealed that external genital organs had signs of age involution,
uterus cervix was not erosive, small amount of bloody discharge came from the
cervical canal. Uterus is of normal size, uterine appendages are unpalpable.
Fornices were deep and painless. What method should be applied for the diagnosis
specification?
A. Separated diagnosic curretage
B. Laparoscopy
C. Puncture of abdominal cavity through posterior vaginal fornix
D. Extensive colposcopy
E. Culdoscopy

39
4. A 51-year-old patient complains of having intensive bloody discharges from
vagina for 15 days after delay of menstruation for 2,5 months. In anamnesis:
disorders of menstrual function during a year, at the same time she felt extreme
irritability and had sleep disorders. US examination results: uterus corresponds
with age norms, appendages have no pecularities, endometrium is 14 mm thick.
What is the doctor's tactics?
A. Diagnostic curettage of uterine cavity
B. Conservative treatment of bleeding
C. Hysterectomy
D. Supravaginal amputation of uterus without appendages
E. TORCH-infection test
5. A 54-year-old female patient consulted a doctor about bloody discharges from
the genital tracts after 2 years of amenorrhea. USI and bimanual examination
revealed no genital pathology. What is the tactics of choice?
A. Fractional biopsy of lining of uterus and uterine mucous membranes
B. Styptic drugs
C. Contracting drugs
D. Estrogenic haemostasia
E. Hysterectomy

6. A 30-year-old female patient has been delivered to the gynaecological


department with complaints of acute pain in the lower abdomen and body
temperature 38,8°C. In history: sexual life out of wedlock and two artificial
abortions. Gynaecological examination reveals no changes of uterine. The
appendages are enlarged and painful on both sides. Vaginal discharges are purulent
and profuse. What study is required to confirm a diagnosis?
A. Bacteriological and bacterioscopic analysis
B. Hysteroscopy
C. Curettage of uterine cavity
D. Colposcopy
E. Laparoscopy

7. A woman complains of having slight dark bloody discharges and mild pains in
the lower part of abdomen for several days. Last menses were 7 weeks ago. The
pregnancy test is positive. Bimanual investigation: the body of the uterus indicates
for about 5-6 weeks of pregnancy, it is soft, painless. In the left appendage there is
a retort-like formation, 7x5 cm large, mobile, painless. What examination is
necessary for detection of fetus localization?
A. Ultrasound
B. Hysteroscopy

40
C. Hromohydrotubation
D. Colposcopy
E. Cystoscopy

8. In the woman of 24 years about earlier normal menstrual function, cycles


became irregular, according to tests of function diagnostics - anovulatory. The
contents of prolactin in blood is boosted. Choose the most suitable investigation:
A. Computer tomography of the head
B. Determination of the level of gonadotropins
C. USI of organs of small pelvis
D. Progesterone assay
E. Determination of the contents of testosteron-depotum in blood serum

9. A 40 year old woman has a selfdetected hard breast mass. The procedure of
choice for confirming the diagnosis is:
A. Excision biopsy
B. Mammography
C. Thermography
D. Ultrasonography
E. Aspiration biopsy with cytology

10. A 13 year old girl consulted the school doctor on account of moderate bloody
discharge from the genital tracts, which appeared 2 days ago. Secondary sexual
characters are developed. What is the most probable cause of bloody discharge?
A. Menarche
B. Juvenile hemorrhage
C. Haemophilia
D. Endometrium cancer
E. Werlhof's disease

11. A 19-year-old woman is brought to the emergency department because of


severe lower abdominal pain. Over the past 24 hours, she has had several episodes
of severe abdominal pain lasting for 15 to 20 minutes and then resolving. With the
episodes of pain, she has nausea, vomiting, and diaphoresis. Her temperature is
37.7 C, blood pressure is 114/78 mm Hg, pulse is 110/min, and respirations are
14/min. Her lower abdomen is bilaterally tender, more on the left than the right.
Pelvic examination is somewhat limited because of the patient’s inability to
tolerate it, but there is the suggestion of a left adnexal mass. Urine hCG and
urinalysis are negative. Which of the following is the most appropriate next step in
diagnosis?
A. Pelvic ultrasound

41
B. Abdominal x-ray
C. CT scan
D. MRI
E. Culdocentesis

12. An 18-year-old woman has a 2-cm, firm, rubbery mass in the upper outer
quadrant of her left breast. It has been present for at least 3 or 4 months. The mass
is easily movable, not tender, and otherwise asymptomatic. Which of the following
is the most appropriate initial step in management?
A. Clinical observation
B. Sonogram
C. Mammogram
D. Incisional biopsy
E. Excisional biopsy

2. Inflammatory diseases of female genital organs


1. A 22-year-old woman complains of itching and profuse discharge from her
genital tracts. The condition developed 10 days ago after a sexual contact.
Bacterioscopy of a discharge sample detected trichomonads. What drug should be
prescribed for treatment in this case?
A. Metronidazole
B. Ampicillin
C. Erythromycin
D. Zovirax (Acyclovir)
E. Valcyclovir

2. A 17-year-old girl has made an appointment with the doctor. She plans to begin
her sex life. No signs of gynecological pathology were detected. In the family
history there was a case of cervical cancer that occurred to the patient’s
grandmother. The patient was consulted about the maintenance of her reproductive
health. What recommendation will be the most helpful for prevention of invasive
cervical cancer?
A. Vaccination against human papillomavirus (HPV)
B. Vitamins, calcium, omega-3
C. Immunomodulators
D. Antiviral and antibacterial drugs
E. Timely treatment of sexually transmitted diseases

3. A 37-year-old woman complains of acute pain in the genital area, swelling of


the labia, pain when walking. Objectively: body temperature is 38,7oC, Ps-
98/min. In the interior of the right labia there is a dense, painful tumor-like

42
formation 5,0x4,5 cm in size, the skin and mucous membrane of genitals are
hyperemic, there is profuse foul-smelling discharge. What is the most likely
diagnosis?
A. Acute bartholinitis
B. Labial furuncle
C. Acute vulvovaginitis
D. Bartholin gland cyst
E. Carcinoma of vulva

4. An 18-year-old woman complains of pain in her lower abdomen, profuse


purulent discharge from the vagina, temperature rise up to 37,8oC. Anamnesis
states that she had a random sexual contact the day before the signs appeared. She
was diagnosed with acute bilateral adnexitis. On additional examination:
leukocytes are present throughout all vision field, bacteria, diplococci with
intracellular and extracellular position. What is the most likely agent in the given
case?
A. Neisseria gonorrhoeae
B. Escherichia coli
C. Chlamydia trachomatis
D. Trichomona vaginalis
E. Staphylococcus aureus

5. A 6-year-old girl came to a general practitioner with her mother. The child
complains of burning pain and itching in her external genitalia. The girl was taking
antibiotics the day before due to her suffering from acute bronchitis. On
examination: external genitalia are swollen, hyperemic, there is white deposit
accumulated in the folds. The most likely diagnosis is:
A. Candidal vulvovaginitis
B. Trichomoniasis
C. Nonspecific vulvitis
D. Helminthic invasion
E. Herpes vulvitis

6. A 21-year-old female patient consulted a gynecologist about itching, burning,


watery vaginal discharges with a fish-like smell. Speculum examination revealed
that the cervical and vaginal mucosa was of a normal pink color. Vaginal
examination revealed no alterations of the uterus and appendages. Gram-stained
smears included clue cells. What is the most likely pathology?
A. Bacterial vaginosis (gardnerellosis)
B. Chlamydiosis
C. Gonorrhea

43
D. Trichomoniasis
E. Candidiasis

7. On the fifth day after a casual sexual contact a 25-year-old female patient
consulted a doctor about purulent discharges from the genital tracts and itch.
Vaginal examination showed that vaginal part of uterine cervix was hyperemic and
edematic. There was an erosive area around the external orifice of uterus. There
were mucopurulent profuse discharges from the cervical canal, uterine body and
appendages exhibited no changes. Bacterioscopic examination revealed bean-
shaped diplococci that became red after Gram's staining. What is the most likely
diagnosis?
A. Acute gonorrheal endocervicitis
B. Trichomonal colpitis
C. Candidal vulvovaginitis
D. Clamydial endocervicitis
E. Bacterial vaginism

8. A 28-year-old patient has been admitted to the gynecological department three


days after a casual coitus. She complains about pain in her lower abdomen and
during urination, profuse purulent discharges from the vagina, body temperature
rise up to 37,8oC. The patient was diagnosed with acute bi- lateral adnexitis.
Supplemental examination revealed: the 4th degree of purity of the vaginal
secretion, leukocytes within the whole visual field, diplococcal bacteria located
both intra- and extracellularly. What is the etiology of acute adnexitis in this
patient?
A. Gonorrheal
B. Colibacterial
C. Chlamydial
D. Trichomonadal
E. Staphylococcal

9. A 42-year-old woman comes to the physician because of vaginal itch and


discharge, dysuria, and dyspareunia. These symptoms have been steadily
worsening over the past 3 days. Pelvic examination reveals an erythematous vagina
and a thin, green, frothy vaginal discharge with a pH of 6. Microscopic
examination of the discharge demonstrates the presence of a pear-shaped, motile
organism. Which of the following is the most likely pathogen?
A. Candida albicans
B. Gardnerella vaginalis
C. Herpes simplex virus

44
D. Treponema pallidum
E. Trichomonas vaginalis

10. A 22-year-old woman, gravida 2, para 1, comes to the physician for her first
prenatal visit. She had a previous full-term, normal vaginal delivery 2 years ago.
She has no medical problems and has never had surgery. She takes no medications
and has no known drug allergies. Pelvic examination reveals a mucopurulent
cervical discharge, no cervical motion tenderness, and an 8-week-sized, nontender
uterus. A cervical swab is performed. Two days later, the laboratory calls to notify
the physician that the patient is positive for Chlamydia trachomatis. Which of the
following is the most appropriate pharmacotherapy?
A. Ceftriaxone
B. Erythromycin
C. Metronidazole
D. Penicillin
E. Tetracycline

11. A 22-year-old woman comes to the physician with her husband because of
vaginal irritation and a malodorous vaginal discharge. Her symptoms started 4
days ago. She also notes pain with intercourse and dysuria. Pelvic examination
reveals vaginal and cervical erythema and a copious greenish, frothy discharge.
The pH of this discharge is 6.0. A wet preparation is done with normal saline,
which shows numerous flagellated organisms that are slightly larger than the
surrounding white blood cells. Which of the following is the most appropriate
management?
A. Do not treat the patient or her partner
B. Treat only the patient with metronidazole
C. Treat the patient and her partner with metronidazole
D. Treat only the patient with penicillin
E. Treat the patient and her partner with penicillin

12. A 33-year-old woman presents to the physician because of a malodorous


vaginal discharge that has been present for the past 3 days. She has no vaginal or
vulvar irritation, and has no urinary complaints. Pelvic examination demonstrates a
copious, gray discharge with a pH of 5.0. When 1 drop of potassium hydroxide
(KOH) is added to a sample of the discharge there is an intense amine odor. A
normal saline wet preparation is performed that demonstrates epithelial cells whose

45
borders and nuclei are obscured by the presence of bacteria. Which of the
following is the most likely pathogen?
A. Candida albicans
B. Chlamydia trachomatis
C. Gardnerella vaginalis
D. Lactobacillus species
E. Trichomonas vaginalis

3. Menstrual disorders
1. A 15-year-old adolescent girl came the the gynecologist with complaints of
painful menstruations that are accompanied by nausea, vomiting, and dizziness.
Her menarche was at 12. Menstruations became painful since she was 14, remain
regular. What treatment should be prescribed in this case?
A. Analgesics, antispasmodics, antiprostaglandine therapy
B. Antiinflammatory treatment only
C. Antihemorrhagic agents
D. Antiandrogen therapy
E. Vitamin supplements

2. A 46-year-old woman came to a maternity clinic with complaints of moderate


blood discharge from the vagina, which developed after the menstruation delay of
1,5 months. On vaginal examination: the cervix is clean; the uterus is not enlarged,
mobile, painless; appendages without changes. Make the diagnosis:
A. Dysfunctional uterine bleeding
B. Adenomyosis
C. Ectopic pregnancy
D. Submucous uterine myoma
E. Cancer of the uterine body

3. A 14-year-old girl has been delivered to a gynecological department with


complaints of profuse blood discharge from her genital tract for 2 weeks.
Anamnesis: menstruation since 13, irregular, painful, profuse; the last one was 2
months ago. Objectively: pale skin and mucosa, BP- 100/60 mm Hg, Hb-108 g/l.
The abdomen is soft and painless on palpation. Rectal examination revealed no
pathologies of reproductive organs. What condition is it?
A. Juvenile uterine hemorrhage (Dysfunctional)
B. Hypomenstrual syndrome

46
C. Inflammation of uterine appendages (Pelvic inflammatory disease)
D. Pelviperitonitis
E Endometritis

4. A 25-year-old woman complains of menstruation retention lasting for 3 years.


The patient explains it by a difficult childbirth complicated with profuse
hemorrhage, weight loss, brittleness and loss of hair, loss of appetite, depression.
Objective examination reveals no pathologic changes of uterus and uterine
appendages. What pathogenesis is characteristic of this disorder?
A. Decreased production of gonadotropin
B. Hyperproduction of estrogen
C. Hyperproduction of androgen
D. Decreased production of progesterone
E. Hyperproduction of prolactin

5.A 20-year-old female consulted a gynecologist about not having menstrual


period for 7 months. History abstracts: early childhood infections and frequent
tonsillitis, menarche since 13 years, regular monthly menstrual cycle of 28 days,
painless menstruation lasts 5-6 days. 7 months ago the patient had an emotional
stress. Gynecological examination revealed no alterations in the uterus. What is the
most likely diagnosis?
A. Secondary amenorrhea
B. Primary amenorrhea
C. Algomenorrhea
D. Spanomenorrhea
Е. Cryptomenorrhea.

6. A 28-year-old patient complains of infertility. The patient has been married for 4
years, has regular sexual life and does not use contraceptives but has never got
pregnant. Examination revealed normal state of the genitals, tubal patency. Basal
body temperature recorded over the course of 3 consecutive menstrual cycles
appeared to have a single phase. What is the most likely cause of infertility?
A. Anovulatory menstrual cycle
B. Immunological infertility
C. Genital endometriosis
D. Chronic salpingoophoritis
E. Ovulatory menstrual cycle

47
7. A 28-year-old patient complains of profuse, painful and prolonged menstruation.
Before and after the menstrual period there is spotting lasting for 4-6 days. Vaginal
examination reveals that the uterus is enlarged corresponding to 5-6 weeks of
pregnancy, has limited mobility, is painful. Appendages are not palpable. On the
15th day of the menstrual cycle, the uterus was of normal size, painless. On
account of stated problems and objective examination the patient has been
diagnosed with internal endometriosis. Which drug should be used for the efective
treatment of this patient?
A. Duphaston
B. Synoestrolum
C. Parlodel
D. Ovidon

8. A 13-year-old girl was admitted to the gynecological department with heavy


bleeding, which appeared after a long delay of menstruation. Shortly before, the
girl suffered a serious psychotrauma. Her menarche occurred at the age of 11, she
has a 30-day cycle with 5 to 6 days of moderate, painless bleeding. The patient is
somatically healthy, of normosthenic constitution with height of 160 cm, weight of
42 kg. The patient is pale. Rectoabdominal examination revealed that the uterus
was of normal size and consistency, anteflexio-versio, the appendages were not
changed. What is the most likely diagnosis?
A. Juvenile bleeding
B. Ovarian cyst
C. Hysteromyoma
D. Girl is healthy
E. Amenorrhea

9. A 55-year-old patient whose menstruation stopped 5 years ago complains of


vaginal dryness, frequent and painful urination. Gynecologist revealed signs of
atrophic colpitis. Urine analysis revealed no peculiarities. Which locally acting
product will provide the proper therapeutic effect?
A. Vaginal suppositories "Ovestin"
B. Vaginal tablets "Tergynan"
C. Vaginal cream "Meratin Combi"
D. Vaginal gel "Metronidazole"
Е. Vaginal cream "Dalacin

48
10. A 14 year old girl complains of profuse bloody discharges from genital tracts
during 10 days after suppresion of menses for 1,5 month. Similiar bleedings recur
since 12 years on the background of disordered menstrual cycle. On rectal
examination: no pathology of the internal genitalia. In blood: Hb - 70 g/l, RBC-2,3
• 1012/l, Ht - 20. What is the most probable diagnosis?
A. Juvenile bleeding, posthemorrhagic anemia
B. Werlholf's disease
C. Polycyst ovarian syndrome
D. Hormonoproductive ovary tumor
E. Incomplete spontaneous abortion

11. A 24-year-old woman, gravida 2, para 2, comes to the physician for a yearly
physical and birth control counseling. She is currently using the rhythm method of
birth control, but has heard that this method has a high failure rate and would like
to try a different method. Several of her friends use the intrauterine device (IUD),
and she is wondering whether she could also use this method. Past medical history
is significant for eczema. Past surgical history is significant for a right ovarian
cystectomy 2 years ago. Past gynecologic history is significant for multiple
episodes of Chlamydia cervicitis and two episodes of pelvic inflammatory disease
(PID), the most recent episode occurring 1 year ago. She takes acetaminophen for
occasional tension headaches. She is allergic to penicillin. She smokes one- half
pack of cigarettes per day. Physical examination is unremarkable. Which of the
following would be the best recommendation for this patient regarding her birth
control method?
A. “The IUD is absolutely contraindicated.”
B. “The IUD is recommended.”
C. “The IUD is recommended if cervical cultures are negative.”
D. “The oral contraceptive pill is absolutely contraindicated.”
E. “The rhythm method is recommended.”

12. A 14-year-old girl comes to the physician because of lower abdominal


cramping. This cramping starts a few hours before, and lasts through, her menses,
and then resolves completely. The cramping is primarily in the lower abdomen but
also radiates to the back and thighs. She first noted this cramping approximately 6
months after her first menstrual period at age 12. She is not sexually active.
Physical examination is unremarkable, including a normal pelvic examination. A
pregnancy test is negative. Which of the following is the most appropriate next
step in management?
A. Trial of nonsteroidal anti-inflammatory drugs (NSAIDs)

49
B. Trial of antibiotics
C. GnRH agonist therapy
D. Laparoscopy
E Laparotomy
13. A 14-year-old girl comes to the physician complaining of pelvic pain each
month. She states that approximately every 30 days she develops crampy lower
abdominal pain that resolves after a day or two. She has never had a menstrual
period. Examination shows normal development of the breasts and the presence of
axillary and pubic hair. Pelvic examination demonstrates a vaginal bulge. Rectal
examination reveals a mass anterior to the rectum. Urine hCG is negative. Which
of the following is the most likely diagnosis?
A. Colon cancer
B. Ectopic pregnancy
C. Endometriosis
D. Imperforate hymen
E. Vaginal cancer

4. Neuroendocrine syndromes
1. A 16-year-old girl has primary amenorrhea, no pubic hair growth, normally
developed mammary glands; her genotype is 46 XY; uterus and vagina are
absent. What is your diagnosis?
A. Testicular feminization syndrome
B. Mayer-Rokitansky-Kuster-Hauser syndrome
C. Cushing syndrome
D. Sheehan syndrome
E. Cushing disease

2. A woman came to the general practitioner with complaints of fatigability,


significant weight loss, weakness, and loss of appetite. She has been presenting
with amenorrhea for the last 8 month. One year ago she gave birth to a live full-
term child. Blood loss during delivery was 2 liters. The woman received blood
transfusion and blood components. What is the most likely diagnosis?
A. Sheehan’s syndrome (postpartum hypopituitarism)
B. Stein-Leventhal syndrome (polycystic ovary)
C. Turner’s syndrome
D. Homologous blood syndrome
E. Somatoform autonomic dysfunction

50
3. A 30-year-old woman complains of milk discharge from her breasts and no
menstruation for the last 5 months. One physiologic childbirth was 4 years ago.
There are no maldevelopments of mammary glands. Bimanual examination
revealed diminished uterus and normal sized ovaries. MRI- scan shows no brain
pathologies. Thyroid- stimulating hormone is within normal limits. Serum
prolactin is high. What is the most likely diagnosis?
A. Hyperprolactinemia
B. Hypothyroidism
C. Polycystic ovaries
D. Pituitary adenoma
E. Sheehan’s syndrome (postpartum hypopituitarism)

4. A 29-year-old woman came to a gynecologist with complaints of irritability,


tearfulness, headache, nausea, occasional vomiting, pain in the heart area,
tachycardia attacks, memory impairment, meteorism. These signs appear 6 days
before menstruation and disappear the day before menstruation or during its first 2
days. On vaginal examination: the uterus and uterine appendages are without
alterations. What diagnosis is the most likely?
A. Premenstrual syndrome
B. Algodismenorrhea
C. Ovarian apoplexy
D. Genital endometriosis
E. Neurosis

5. A 30-year-old woman complains of amenorrhea that lasts for 2 years after she
has given birth, loss of hair and body weight. The labor was complicated with
hemorrhage caused by uterine hypotonia. Objectively the patient is of asthenic
type, her external genitalia are hypoplastic, the uterine body is small in size and
painless. No uterine appendages can be detected. What is the most likely
diagnosis?
A. Sheehan's syndrome (postpartum hypopituitarism)
B. Ovarian amenorrhea
C. Turner's syndrome
D. Ovarian exhaustion syndrome
E. Galactorrhea-amenorrhea syndrome

51
6. A 28-year-old woman complains of increased intervals between menstruations,
up to 2 months, and hirsutism. Gynecological examination revealed the following:
ovaries are enlarged, painless, and dense; no alterations of the uterus. US of the
lesser pelvis: ovaries are 4-5 cm in diameter, with numerous enlarged follicles on
the periphery. X-ray of the skull base: sellar region is widened. What is the most
likely diagnosis?
A. Stein-Leventhal syndrome (polycystic ovarian syndrome)
B. Algodismenorrhea
C. Sheehan syndrome (postpartum hypopituitarism)
D. Premenstrual syndrome
E. Morgagni-Stewart-Morel syndrome (metabolic craniopathy)

7. A 22-year-old woman complains of amenorrhea for 8 months. Anamnesis states


that menarche occured at the age of 12,5. Since the age of 18 the patient has a
history of irregular menstruation. The patient is nulli-gravida. The mammary
glands are developed properly, nipples discharge drops of milk when pressed.
Hormone test: prolactin level is 2 times higher than normal. CT reveals a bulky
formation with diameter of 4 mm in the region of sella. What is the most likely
diagnosis?
A. Pituitary tumour
B. Lactational amenorrhea
C. Stein-Leventhal syndrome (polycystic ovary syndrome)
D. Sheehan's syndrome (postpartum hypopituitarism)
E. Cushing's disease

8. A 16-year-old girl has primary amenorrhea, no pubic hair growth,


normally developed mammary glands; her genotype is 46 XY; uterus and
vagina are absent. What is your diagnosis?
A. Testicular feminization syndrome
B. Mayer-Rokitansky-Kuster-Hauser syndrome
C. Cushing's syndrome
D. Sheehan syndrome
E. Cushing's disease

9. A 26-year-old woman has attended maternity center complaining of her inability


to become pregnant despite 3 years of regular sex life. Examination revealed the

52
following: increased body weight; male-type pubic hair; excessive pilosis of
thighs; ovaries are dense and enlarged; basal body temperature is monophasic. The
most likely diagnosis is:
A. Ovaries sclerocystosis
B. Inflammation of uterine appendages
C. Adrenogenital syndrome
D. Premenstrual syndrome
E. Gonadal dysgenesis

10. 13 months after the first labor a 24-year-old patient complained of amenorrhea.
Pregnancy ended in Caesarian section because of premature detachment of
normally positioned placenta which resulted in blood loss at the rate of 2000 ml
due to disturbance of blood clotting. Choose the most suitable investigation:
A. Estimation of gonadotropin rate
B. US of small pelvis
C. Progesteron assay
D. Computer tomography of head
E. Estimation of testosteron rate in blood serum

11. A 49-year-old female patient complains of itching, burning in the external


genitals, frequent urination. The symptoms have been present for the last 7 months.
The patient has irregular menstruation, once every 3-4 months. Over the last two
years she has had hot flashes, sweating, sleep disturbance. Examination revealed
no pathological changes of the internal reproductive organs. Complete blood count
and urinalysis showed no pathological changes. Vaginal smear contained 20-25
leukocytes per HPF, mixed flora. What is the most likely diagnosis?
A. Menopausal syndrome
B. Cystitis
C. Trichomonas colpitis
D. Vulvitis
E. Bacterial vaginosis

12. A 38-year-old female patient complains about hot flashes and feeling of intense
heat arising up to 5 times a day, headaches in the occipital region along with high
blood pressure, palpitations, dizziness, fatigue, irritability, memory impairment. 6
months ago the patient underwent extirpation of the uterus with its appendages.
What is the most likely diagnosis?

53
A. Post-castration syndrome
B. Premenstrual syndrome
C. Early pathological menopause
D. Secondary psychogenic amenorrhea
E. Physiological premenopause

5. Endometriosis
1. A 35-year-old woman addressed a gynecological department with complaints of
regular pains in her lower abdomen, which increase during menstruation, and dark-
brown sticky discharge from the genital tracts. On bimanual examination: the
uterine body is slightly enlarged, the appendages are not palpated. Mirror
examination of the uterine cervix reveals bluish spots. What diagnosis is most
likely?
A. Cervical endometriosis
B. Cervical erosion
C. Cervical polyp
D. Cervical cancer
E. Cervical fibroid

2. A 28-year-old woman has bursting pain in the lower abdomen during


menstruation; chocolatelike discharges from vagina are observed. It is known from
the anamnesis that the patient suffers from chronic adnexitis. Bimanual
examination revealed a tumour-like formation of heterogenous consistency 7х7 cm
large to the left from the uterus. The formation is restrictedly movable, painful
when moved. What is the most probable diagnosis?
A. Endometrioid cyst of the left ovary
B. Follicular cyst of the left ovary
C. Fibromatous node
D. Exacerbation of chronic adnexitis
E. Tumour of sigmoid colon

3. A 28-year-old female patient has been admitted to the gynecology department


for abdominal pain, spotting before and after menstruation for 5 days. The disease
is associated with the abortion which she had 2 years ago. Anti-inflammatory
treatment had no effect. Bimanual examination findings: the uterus is enlarged,
tight, painful, smooth. Hysteroscopy reveals dark red holes in the fundus with dark
blood coming out of them. What diagnosis can be made on the grounds of these

54
clinical presentations?
A. Inner endometriosis
B. Polymenorrhea
C. Hypermenorrhea
D. Submucous fibromatous node
E. Dysfunctional uterine bleeding

4. A 42-year-old woman has had hyperpolymenorrhea and progressing


algodismenorrhea for the last 10 years. Gynaecological examination revealed no
changes of uterine cervix; discharges are moderate, of chocolate colour, uterus is
slightly enlarged and painful, appendages are not palpable, the fornices are deep
and painless. What is the most likely diagnosis?
A. Uterine endometriosis
B. Uterine carcinoma
C. Subserous uterine fibromyoma
D. Endomyometritis
E. Adnexal endmetriosis

5. The patient complains of lower abdominal pain, which increases during


menstruation and sexual contacts and irradiates in the vagina. During a bimanual
study - behind the uterus are dense, nodular, painful formations.What is the most
likely diagnosis?
A. Parametritis
B. Perimetritis
C. Adenomyosis
D. Retro-cervical endometriosis
E. Chronic inflammation of the appendages of the uterus

6. A 26-year-old patient turned to a woman's consultation with complaints of lower


abdominal pain, which is increased during menstruation, spotting before and after
menstruation. The disease is associated with an artificial abortion. During
speculum inspection: 5 dark red inclusions on the cervix.Your diagnosis?
A. External endometriosis
B. Polyposis of the cervix
C. Cervical cancer
D. Erosion of the cervix

55
E. Dysplasia of the cervix

6. Anomalies of the position of the internal genital organs


1. A 62-year-old woman complains of difficulties during urination and defecation,
protrusion of a “tumor” from the genital slit, which interferes with walking.
Gynecological status: a “tumor” emerges from the genital slit, in the lower half of
which a hole is visible. The walls of the "tumor" look like dull-shiny dry skin; on
the back wall, pressure sore can be seen up to 2 cm in diameter. In the "tumor", the
uterus is felt, which is completely out of the pudendal cleft. What is the diagnosis?
A. Prolapse of the posterior vaginal wall
B. Prolapse of the front wall of the vagina
C. Incomplete prolapse of the uterus with cervical ulcers
D. Incomplete prolapse of the cervix with pressure sores
E. Complete prolapse of the uterus with decubital ulcer

2. A 45-year-old woman who works as a painter-plasterer, during a medical


examination, an omission of the vaginal wall of an II degree was found.What is the
possible cause of this disease?
A. Violation of bowel function
B. Cervical erosion
C Birth injury
D. Heavy physical work
E. Chronic adnexitis

3. A woman 60 years old of 2 births with a mass of newborns of 4500 and 4800 g
in anamnesis, hard physical work. Complaints of frequent urination, pulling pains
in the lower abdomen, in the lower back. During physical examination, a tumor-
like formation emerges from the pudendal cleft, which is easily repositioned. Put
fingers over the crotch at the vagina prevents the body of the uterus.Your
diagnosis?
A. Incomplete prolapse of the uterus
B. Complete uterus prolapsed
С Gartner cyst
D. Leiomyomatous node, which is born
E. Bartholin cyst gland

4. A 72-year-old woman suffering from atherosclerotic cardiosclerosis with


extrasystolic arrhythmia and grade II heart failure was diagnosed with complete

56
prolapse of the uterus. Sex life does not live. Choose the method of optimal
surgical treatment.
A. Vaginal hysterectomy
B. Middle colporrhaphy
C. Ventroscopy of the uterus
D. Wertheim operation
E. Supravaginal amputation of the uterus without appendage

5. An 85-year-old woman comes to the physician because of pelvic pressure and


the feeling that something is coming out of her vagina. She has a history of
coronary artery disease and is status post a three-vessel coronary artery bypass
graft 10 years ago. She had a cerebrovascular accident 2 years ago that left her
with decreased right-sided sensory and motor function. She takes multiple cardiac
medications. Examination shows morbid obesity. Her uterus is noted to have mild
to moderate prolapse. Which of the following is the most appropriate next step in
management?
A. Oral contraceptive pill
B. Hormone replacement therapy
C. Trial of pessary
D. Vaginal hysterectomy
E. Abdominal hysterectomy

7. Benign tumors
1. A woman complains of muscle weakness and general fatigue, dyspnea, vertigo,
brittleness of her hair and nails, an urge to eat chalk. Anamnesis states uterine
fibroid. Common blood analysis: erythrocytes - 2,8 T/l, Hb- 80 g/l, color index -
0,78, anisocytosis, poikilocythemia, serum iron - 10 mcmol/l. What diagnosis is
most likely?
A. Iron-deficiency anemia
B. B12-deficient anemia
C. Autoimmune hemolytic anemia
D. Aplastic anemia
E. Hypoplastic anemia

2. A 49-year-old patient undergoes regular medical check-up for uterine fi-


bromyoma. Within the last year the uterus has enlarged up to 20 weeks of
gestation. What is the rational way of treatment?
A. Surgical treatment

57
B. Hormonal therapy
C. Further surveillance
D. Embolization of uterine arteries
E. Treatment with prostaglandin inhibitors

3. A 44-year-old woman comes to the physician because of heavy periods. She


states that her periods have gotten increasingly heavy over the past year. She now
has a 6-day menstrual flow and needs to change pads far more frequently than
before. She also complains of constant lower abdominal pressure. She has four
children and does not wish to have any more. Examination shows a 16-week-sized
uterus. Hematocrit is 29%. Endometrial biopsy demonstrates benign endometrial
tissue. Pelvic ultrasound shows an enlarged fibroid uterus with multiple fibroids.
What is the most appropriate next step in management?
A. Hormone replacement therapy
B. Diagnostic laparoscopy
C. Myomectomy
D. Tubal ligation
E. Hysterectomy

4. A woman of 30 years turned to a maternity clinic for a professional examination.


Menstrual function is not impaired. Births, abortions were not. Gynecological
examination of the cervix without epithelial defects, the body of the uterus is
increased to 6-7 weeks of pregnancy, dense, painless. The appendages of the uterus
on both sides are not defined. An ultrasound diagnosis was confirmed - uterine
leiomyoma. What is the doctor's tactic?
A. Hysterectomy
B. Conservative treatment with estrogen
C. Supravaginal amputation of the uterus without appendages
D. Treatment with androgens
E. Clinical observation and conservative treatment

5. A 40-year-old patient complains of heavy menstruation during the year,


accompanied by cramping pains in the lower abdomen. During bimanual
examination : in the cervical canal is determined formation with a diameter of 5
cm thick consistency, the leg of which goes into the uterine cavity. The body of the
uterus is increased to 5-6 weeks of pregnancy, normal consistency, agile, painful.
The appendages are not defined. The discharge from the cervical canal is bloody,
copious. What diagnosis can be assumed?

58
A. Nascent submucous leiomyomatous node
B. Abortion in progress
C. Cervical cancer
D. Cervical leiomyoma
E. Algomenorrhea

6. The patient 15 years old complained of recurrent pain in the lower abdomen,
more on the left. Menstrual function is not impaired. Sex life does not live.
Rectoabdominal examination: the uterus is not changed, the right appendages are
not defined, the left is palpated with the formation of an ovoid shape, 10x11 cm in
size, with a smooth surface, elastic consistense, mobile, painless.
What is the most likely diagnosis?
A. Pyovar left
B. Chronic salpingoophoritis
C. Tumor of the left ovary
D. Ovarian cancer
E. Appendicitis

7. A 49-year-old patient on in the dispensary observation about uterine fibroids.


Over the past year, uterine tumor has increased to 20 weeks of pregnancy.
What is the best treatment?
A. Surgical treatment
B. Hormone therapy
C. Further observation
D. Uterine artery embolization
E. Treatment with prostaglandin inhibitors
8. Clinical manifestation of which ovarian tumor is Meigs syndrome?
A. Dysgerminoma
B. Mibroma
C. Mucinous cystadenomas
D. Granular cell tumor
E. Mature teratoma

9. A 48-year-old patient has complained about the fact that in the last 8–9 months
of menstruation is very abundant, leading to anemization, and working capacity is
impaired. During the 2 years observed by a gynecologist about uterine fibroids.
Objectively: the cervix is cylindrical, clear, the external os is closed. Discharge are
mucous. The arches are deep. The body of the uterus is increased to 9-10 weeks of

59
pregnancy, dense, mobile, painless. The appendages on both sides are not defined.
Parameters are free.What is the most likely diagnosis?
A. Submucous uterine leiomyoma
B. Endometrial cancer
C. Interstitial uterine leiomyoma
D. Uterus Endometriosis
E. Uterine leiomyoma and pregnancy

8. Malignant tumors
1. A 55-year-old woman came to a gynecologist with complaints of leukorrhea and
bloody discharge from the vagina after 5 years of menopause. Anamnesis states no
pregnancies. Bimanual examination: the uterus and uterine appendages are without
changes. During diagnostic curettage of the uterine cavity the physician scraped off
enchephaloid matter. What is the most likely diagnosis in this case?
A. Endometrial carcinoma
B. Adenomyosis
C. Subserous uterine myoma
D. Cervical carcinoma
E. Ovarian carcinoma

2. A 50-year-old female patient complains of aching pain in the lower abdomen.


She has a history of normal menstrual cycle. At the age of 40, the patient
underwent a surgery for gastric ulcer. Examination findings: abdomen is soft, in
the hypogastrium there is a well-defined nodular tumor of limited mobility.
Vaginal examination findings: the cervix is clean, of cylindrical shape. Body of the
uterus cannot be palpated separately. On both sides of the uterus palpation reveals
tight tumors with an uneven surface. The tumors are immobile andl fill the whole
pelvic cavity. What is the most likely diagnosis?
A. Krukenberg tumor
B. Ovarian fibroid
C. Ovarian granulosa cell tumor
D. Bilateral pioovarium
E. Subserous metrofibroma

3. A 48-year-old female has been admitted to the gynecology department for pain
in the lower right abdomen and low back pain, constipations. Bimanual
examination findings: the uterus is immobile, the size of a 10-week pregnancy, has

60
uneven surface. Aspirate from the uterine cavity contains atypical cells. What
diagnosis can be made?
A. Hysterocarcinoma
B. Cervical cancer
C. Metrofibroma
D. Colon cancer
E. Chorionepithelioma

4. A 58-year-old female patient came to the antenatal clinic complaining of bloody


light-red discharges from the genital tracts. Menopause is 12 years. Gynaecological
examination revealed age involution of externalia and vagina; uterine cervix was
unchanged, there were scant bloody discharges from uterine cervix, uterus was of
normal size; uterine appendages were not palpable; parametria were free. What is
the most likely diagnosis?
A. Uterine carcinoma
B. Atrophic colpitis
C. Abnormalities of menstrual cycle of climacteric nature
D. Cervical carcinoma
E. Granulosa cell tumor of ovary

5. A 48 year old female patient complains about contact haemorrhage. Speculum


examination revealed hypertrophy of uterus cervix. It resembles of cauliflower, it
is dense and can be easily injured. Bimanual examination revealed that forni-ces
were shortened, uterine body was nonmobile. What is the most probable diagnosis?
A. Cervical carcinoma
B. Metrofibroma
C. Endometriosis
D. Cervical pregnancy
E. Cervical papillomatosis

6. A 35-year-old woman presents to a physician complaining of irregular menstrual


periods. She had her first menses at age 15 and states that her periods come
irregularly every 2 to 6 months. She has been in a monogamous relationship with
her husband for 15 years; for 10 years they have been trying unsuccessfully to
conceive. She gets yearly Pap smears, which have been normal. Her height is 157.5
cm, and her weight is 90.9 kg. Her temperature is 37.0 C, blood pressure is 118/78
mm Hg, pulse is 80/min, and respirations are 14/min. She has acne, as well as

61
excess hair, on her face and between her breasts. Her abdomen is obese.
Examination is otherwise within normal limits. This patient is at greatest risk for
developing which of the following diseases?
A. Cervical cancer
B. Endometrial cancer
C. Lung cancer
D. Osteoporosis
E. Ovarian cancer

7. A 75-year-old woman comes to the physician complaining of vulvar itch that


has been worsening for the past 2 years. She has had no bleeding from the vagina
since she underwent menopause at the age of 52. She smokes five cigarettes per
day. On physical examination she has a raised, pigmented lesion on the right labia
majora. The rest of her physical examination is unremarkable. Which of the
following is the most appropriate next step in the management of this patient?
A. Prescribe an antibiotic
B. Prescribe an antifungal
C. Prescribe steroid cream
D. Refer to psychiatry
E. Biopsy the lesion

8. A 27-year-old woman at 12 weeks’ gestation presents to the physician for her


first prenatal visit. She has had nausea, but no other complaints. Pelvic
examination shows a bulky cervix with a mass involving the cervix and the upper
vagina. A biopsy of the mass reveals squamous cell carcinoma of the cervix.
Which of the following is the most appropriate management?
A. Expectant management
B. Pap smear in 3 to 6 months
C. Colposcopy in 4 to 6 weeks
D. Cone biopsy
E. Radical hysterectomy

9. Trophoblastic disease
1. A 23-year-old woman came the the gynecologist with complaints of blood
smears from her genital tracts that have been observed for a long time. Her
menstruation has been delayed for 8 weeks. Examination shows the uterine body to
be enlarged up to 14 weeks of pregnancy. US detected a vesicular mole. What

62
tactics should the doctor choose?
A. Curettage of the uterine cavity
B. Hormonal treatment
C. Hemostatic treatment
D. Supravaginal uterine amputation
E. Uterectomy

2. A 28-year-old female patient complains of having haemorrhage from the genital


tracts for 1 month. 6 months ago she had natural delivery and gave birth to a girl
weighing 3100 g. Objectively: the uterus is enlarged to 9-10 weeks, mobile,
painless, of heterogenous consistency. Examination reveals vaginal cyanosis,
anaemia and body temperature rise up to 37,8oC. There is a significant increase in
hCG concentration in the urine. What is your provisional diagnosis?
A. Uterine chorionepithelioma
B. Pregnancy
C. Hydatidiform mole
D. Endometritis
E. Uterine fibromyoma

3. An onset of severe preeclampsia at 16 weeks gestation might be caused by:


A. Hydatidiform mole
B. Anencephaly
C. Twin gestation
D. Maternal renal disease
E. Interventricular defect of the fetus

4. An onset of severe preeclampsia at 16 weeks gestation might be caused by:


A. Anencephaly
B. Maternal renal disease
C. Interventricular defect of the fetus
D. Hydatidiform mole
E. Twin gestation

10. Emergency conditions in gynecology


1. A 28-year-old woman has been delivered to a hospital with acute pain in the
lower abdomen. There was a brief syncope. The delay of menstruation is 2 months.

63
Objectively: the patient has pale skin, BP- 90/50 mm Hg, Ps- 110/min. Lower
abdomen is extremely painful. Vaginal examination reveals uterus enlargement.
Promtov's sign (pain during bimanual gynecological examination) is positive.
Right uterine appendages are enlarged and very painful. Posterior vault hangs over.
What is the most likely diagnosis?
А. Right-sided tubal pregnancy
В. Right ovary apoplexy
С. Acute right-sided salpingoophoritis
D. Pelvioperitonitis
E. Incipient abortion

2. A patient with uterine fibromyoma sized up to 8-9 weeks of pregnancy


consulted a gynaecologist about acute pain in the lower abdomen. Examination
revealed pronounced positive symptoms of peritoneal irritation, high
leukocytosis. Vaginal examination revealed that the uterus was enlarged up to 9
weeks of pregnancy due to the fibromatous nodes, one of which was mobile and
extremely painful. Appendages were not palpable. Discharges were mucous,
coming in moderate amounts. What is the treatment tactics?
A. Urgent surgery (laparotomy)
B. Surveillance and spasmolytic therapy
C. Fractional diagnostic curettage of the uterine cavity
D. Surgical laparoscopy
E. Surveillance and antibacterial therapy

3. A 36-year-old female pesented to a gynecological hospital with a significant


bleeding from the genital tract and a 1-month delay of menstruation. Bimanual
examination revealed soft barrelshaped cervix. Uterus was of normal size,
somewhat softened. Appendages were unremarkable on both sides. Speculum
examination revealed that the cervix was cyanotic, enlarged, with the the external
orifice disclosed up to 0,5 cm. Urine hCG test was positive. What is the most likely
diagnosis?
A. Cervical pregnancy
B. Uterogestation
C. Abortion in progress
D. Threatened miscarriage
E. Ectopic pregnancy

64
4. A 24-year-old female patient complains of acute pain in the lower abdomen that
turned up after a physical stress. She presents with nausea, vomiting, dry mouth
and body temperature 36, 6oC. She has a right ovarian cyst in history. Bimanual
examination reveals that uterus is dense, painless, of normal size. The left fornix is
deep, uterine appendages aren't palpable, the right fornix is contracted. There is a
painful formation on the right of uterus. It's round, elastic and mobile. It is 7x8 cm
large. In blood: leukocytosis with the left shit. What is the most likely diagnosis?
A. Ovarian cyst with pedicle torsion
B. Right-sided pyosalpinx
C. Subserous fibromyoma of uterus
D. Acute metritis
E. Extrauterine pregnancy

5. A 22-year-old female patient complains of dull pain in her right iliac area that
she has been experiencing for a week, morning sickness and gustatory change. She
has a histrory of menstruation delay for 3 weeks. Objectively: AP-80/50 mm Hg,
pulse is 78 bpm, body temperature is 37°C. Bimanual examination reveals that
uterus is enlarged, soft, mobile and painless. Uterine appendages are palpable on
the right, there is a dense, elastic and moderately painful formation 3x4 cm large.
What is the most likely diagnosis?
A. Progressing fallopian pregnancy
B. Interrupted fallopian pregnancy
C. Right ovarian cyst
D. Uterogestation
E. Acute appendicitis

6. A 40-year-old female patient has been observing profuse menses accompanied


by spasmodic pain in the lower abdomen for a year. Bimanual examination
performed during menstruation revealed a dense formation up to 5 cm in diameter
in the cervical canal. Uterus is enlarged up to 5-6 weeks of pregnancy, movable,
painful, of normal consistency. Appendages are not palpable. Bloody discharges
are profuse. What is the most likely diagnosis?
A. Nascent submucous fibromatous node
B. Abortion in progress
C. Cervical carcinoma
D. Cervical myoma
E. Algodismenorrhea

65
7. A 24-year-old woman presents to the emergency department complaining of
right lower quadrant pain and vaginal spotting. Her last menstrual period was 5
weeks ago. Her temperature is 37.0 C, blood pressure is 112/70 mm Hg, pulse is
74/min, and respirations are 14/min. The abdomen is soft and nontender. Pelvic
examination reveals scant blood in the vagina, a closed cervical os, no pelvic
masses, and right pelvic tenderness. Her leukocyte count is 8000/mm3, hematocrit
is 38%, and a platelet count is 250,000/mm3. Which of the following is the most
appropriate step next in diagnosis?
A Serum hCG
B Serum TSH
C Abdominal x-ray
D Abdominal/pelvic CT
E Laparoscopy

8. A 32-year-old woman presents to the emergency department complaining of


heavy vaginal bleeding. Her temperature is 37.0 C, blood pressure is 80/50 mm
Hg, pulse is 110/min, and respirations are 18/min. Her abdomen is soft, nontender
and nondistended. Her pelvic examination reveals approximately 200 mL of
clotted blood in the vagina, an open cervical os with tissue protruding from it, and
a 10-week-sized, nontender uterus. Leukocyte count is 9000/mm3, hematocrit is
22%, and platelet count is 275,000/mm3. Quantitative hCG is 100,000 mlU/L
(normal: 5-200,000 mlU/L). Pelvic ultrasound shows echogenic material within the
uterine cavity consistent with blood or tissue, no adnexal masses, and no free fluid.
No viable pregnancy is seen. Which of the following is the most appropriate next
step in management?
A. Discharge to home
B. Culdocentesis
C. Dilation and evacuation
D. Laparoscopy
E. Laparotomy

11. Mammary glands diseases


1. A 25-year-old woman during selfexamination detected a tumor in the upper
external quadrant of her right mammary gland. On palpation: painless, dense,
mobile growth 2 cm in diameter is detected in the mammary gland; no changes in
the peripheral lymph nodes are observed. On US of the mammary glands: in the
upper external quadrant of the right mammary gland there is a space-occupying

66
lesion of increased echogenicity 21x18 mm in size. The most likely diagnosis is:
A. Fibrous adenoma
B. Breast cyst
C. Diffuse mastopathy
D. Breast cancer
E. Mastitis

2. A 45-year-old woman came to the maternity clinic with complaints of periodical


pains in her mammary glands that start 1 day before menstruation and stop after
the menstruation begins. Palpation of the mammary glands detects diffuse nodes
predominantly in the upper outer quadrants. What is the most likely diagnosis?
A. Fibrocystic mastopathy
B. Breast cancer
C. Mastitis
D. Hyperprolactinemia
E. Breast cyst

3. A 25-year-old woman has a self-detected tumor in the upper outer quadrant of


her right breast. On palpation there is a painless firm mobile lump up to 2 cm in
diameter, peripheral lymph nodes are without alterations. In the upper outer
quadrant of the right breast ultrasound revealed a massive neoplasm with increased
echogenicity sized 21x18 mm. What is the most likely diagnosis?
A. Fibroadenoma
B. Lactocele
C. Diffuse mastopathy
D. Mammary cancer
E. Mastitis

4. During a regular check-up of a 50-year-old woman a tumor was detected in her


right mammary gland. The tumor is 5 cm in diameter, dense, without clear
margins. The skin over the tumor resembles lemon rind, the nipple is inverted. The
lymph node can be palpated in the axillary region. What diagnosis is most likely?
A. Breast cancer
B. Lacteal cyst
C. Diffuse mastopathy
D. Mastitis
E. Breast lipoma

67
5. After examination a 46-year-old patient was diagnosed with left breast cancer
T2N2M0, clinical group II-a. What will be the treatment plan for this patient?
A. Radiation therapy + operation + chemotherapy
B. Operation only
C. Operation + radiation therapy
D. Radiation therapy only
E. Chemotherapy only

6. During the breast self-exam a 37-year-old female patient revealed a lump in the
lower inner quadrant of her left breast. Palpation confirms presence of a mobile
well-defined neoplasm up to 2 cm large. Peripheral lymph nodes are not changed.
What is the way of further management?
A. Ultrasound examination of breasts, mammography, fine-needle aspiration
biopsy
B. Anti-inflammatory therapy, physiotherapy
C. Radical mastectomy
D. Ultrasound monitoring of genitals during the entire course of antiestrogens
therapy, systemic enzyme therapy, phytotherapy
E. Case follow-up

7. A 68-year-old patient consulted a doctor about a tumour in her left mammary


gland.Objectively: in the upper internal quadrant of the left mammary gland there
is a neoplasm up to 2,5 cm in diameter, dense, uneven, painless on palpation.
Regional lymph nodes are not enlarged. What is the most likely diagnosis?
A. Cancer
B. Cyst
C. Fibroadenoma
D. Mastopathy
E. Lipoma

8. A 40 year old woman has changes of mammary gland. What are the most often
symtomps that precede the malignizati-on?
A. Skin induration with inverted nipple
B. Painful movable induration
C. Painless movable induration

68
D. Bloody discharges from the nipple
E. Pure discharges from the nipple

9. A 64-year-old woman undergoes left radical mastectomy for breast cancer. A 4-


cm infiltrating ductal carcinoma is found on pathologic examination. Four of 20
axillary lymph nodes are positive for malignancy. Neoplastic cells are
immunoreactive for estrogen and progesterone receptors. No evidence of
metastatic disease is found on bone scanning with 99mTc-labeled phosphate or
chest x-ray films. The patient receives appropriate radiation therapy and multidrug
chemotherapy. Which of the following is the most appropriate adjunctive therapy
in this setting?
A. Danazol
B. Ethinyl estradiol
C. Megestrol acetate
D. Medroxyprogesterone acetate
E. Natural progesterone
F. Tamoxifen

10. An otherwise healthy, 65-year-old woman comes to the physician because of


bloody discharge from the right nipple for 2 weeks. On examination, no retraction,
erosion, or other abnormal change is present. Palpation reveals an ill-defined, 1-cm
nodule located deep in the right areola. Which of the following is the most
appropriate next step in diagnosis?
A. Cytologic examination of nipple discharge
B. Mammography alone
C. Ultrasonography
D. Biopsy under mammographic localization
E. Mammography followed by fine-needle cytology

12. Sterility
1. A 32-year-old woman addressed a maternity clinic with complaints of infertility
that has been lasting for 7 years. Her menstrual cycle occurs in two phases.
Hysterosalpingography reveals obstruction of the uterine tubes in the ampullar
areas, an adhesive process in the small pelvis can be observed. What treatment is
most advisable in this case?
A. Laparoscopy
B. Laparotomy

69
C. Tubectomy
D. Adnexectomy
E. Hydrotubation

2. A 31-year-old female patient complains of infertility, amenorrhea for 2 years


after the artificial abortion that was complicated by endometritis. Objectively:
examination of the external genitalia revals no pathology, there is female pattern of
hair distribution. According to the functional tests, the patient has biphasic
ovulatory cycle. What form of infertility is the case?
A. Uterine
B. Ovarian
C. Pituitary
D. Hypothalamic
E. Immunological

3. A female patient complains of being unable to get pregnant for 5 years. A


complete clinical examination brought the following results: hormonal function is
not impaired, urogenital infection hasn't been found, on hysterosalpingography
both tubes were filled with the contrast medium up to the isthmic segment,
abdominal contrast was not visualized. The patient's husband is healthy. What
tactics will be most effective?
A. In-vitro fertilization
B. Insemination with husband's sperm
C. ICSI within in-vitro fertilization program
D. Hydrotubation
E. Laparoscopic tubal plasty

4. Which of the methods of examination is the most informative in the diagnostics


of a tube infertility?
A. Laparoscopy with chromosalpingoscopy
B. Pertubation
C. Hysterosalpingography
D. Transvaginal echography
E. Bicontrast pelviography

5. A 29-year-old woman comes to the physician complaining of persistent

70
dysmenorrhea and dyspareunia. Both began approximately 4 years ago. The patient
has tried nonsteroidal anti-inflammatory drugs (NSAIDs) and has been on the oral
contraceptive pill (OCP) for a few years without relief. The patient is brought to
the operating room for laparoscopy, during which multiple lesions along her
anterior and posterior cul-de-sac are noted. Many of these lesions appear like “gun-
powdei burns,” whereas others are reddish or bluish. The patient also has
thickening of her uterosacral ligaments with nodularity. In addition to
dysmenorrhea and dys pareunia, which of the following conditions does this
patient most likely have?
A. Basal cell carcinoma
B.Infertility
C.Lenghty menstrual cycles
D.Lung cancer
E.Menorrhagia

6. A woman 37 years old applied to the antenatal clinic with complaints of


infertility for 7 years. The menstrual cycle is two-phase. According to
hysterosalpingography, fallopian tubes are impassable in ampullary areas, signs of
adhesions in the pelvis. What method of treatment will be most appropriate in this
case?
A. Hydrotubation
B. Laparoscopy
C. Tubectomy
D. Adnexectomy
E. Laparotomy
7. A woman of 32 years in the past underwent two operations for ectopic
pregnancy, both fallopian tubes are removed. Appealed to the consultation with the
question of what can be done to come pregnancy?
A. Surrogate motherhood
B. Insemination with husband's sperm
C. In Vitro Fertilization
D. Artificial insemination with donor sperm
E. Ovulation induction

8. A 27-year-old female visited female consultation with infertility. Married during


4 years, is not protected from pregnancy. During examination was established:
development of genitals without deviations from the norm. The passability of the

71
fallopian tubes is not broken. The basal temperature for three menstrual cycles
single phase. What is the most likely cause of infertility?
A. Genital endometriosis
B. Chronic adnexitis
C. Anomalies of genital development
D. Anovulatory menstrual cycle
E. Immunological infertility

13. Operative gynecology


1. The gynecology unit received a patient with uterine bleeding that started 6 hours
after induced abortion at the term of 11-12 weeks. Objectively the skin is pale,
pulse is 100/min., blood pressure is 100/70 mm Hg. On vaginal examination the
uterus is painless, its enlargement corresponds to the 10th week of pregnancy;
uterine cervix is dilated enough to let in one finger, there are fragments of the ferti-
lized ovum. What actions should be taken next:
A. Urgent repeated curettage of the uterine cavity
B. Uterotonic drugs
C. Treatment for acute anemia
D. Antibacterial agents
E. Prescribe rest and continue to monitor the patient’s condition

2. A patient with fibromyoma of uterus sized up to 8-9 weeks of pregnancy


consulted a gynaecologist about acute pain in the lower abdomen. Examination
revealed pronounced positive symptoms of peritoneal irritation, high leukocytosis.
Vaginal examination revealed that the uterus was enlarged corresponding to 9
weeks of pregnancy due to the fi-bromatous nodes, one of which was mobile and
extremely painful. Appendages were not palpable. There were moderate mucous
discharges. What is the optimal treatment tactics?
A. Urgent surgery (laparotomy)
B. Surveillance and spasmolytic therapy
C. Fractional diagnostic curettage of the uterine cavity
D. Surgical laparoscopy
E. Surveillance and antibacterial therapy

3. A 30-year-old woman complains of irregular copious painful menstruations,


pain irradiates to the rectum. Anamnesis states 10-year-long infertility. On

72
bimanual examination: uterus is of normal size; uterine appendages on the both
sides are corded, with rectricted mobility,painful; there are dense nodular painful
growths detected in the posterior fornix. A doctor suspects endometriosis. What
method allows to verify this diagnosis?
A. Laparoscopy
B. Diagnostic curettage of uterine cavity
C. Paracentesis of posterior fornix
D. Uterine probing
E. Hysteroscopy

4. A 29 year old patient underwent surgical treatment because of the benign serous
epithelial tumour of an ovary. The postoperative period has elapsed without
complications. What is it necessary to prescribe for the rehabilitational period:
A. Hormonotherapy and proteolytic enzymes
B. Antibacterial therapy and adaptogens
C. Lasertherapy and enzymotherapy
D. Magnitotherapy and vitamin therapy
E. The patient does not require further care

5. A 42-year-old woman comes to the physician because of irregular vaginal


bleeding. She has a normal menstrual period every 29 days that lasts 3-4 days.
Then, a few days after the cessation of her normal menses, she has a “second
period” that lasts 1-2 days. Physical examination is unremarkable, including a
normal pelvic examination. Urine hCG is negative. Endometrial biopsy suggests
the presence of an endometrial polyp. Pap smear is within normal limits. Office
hysteroscopy reveals a 2-3 cm endometrial polyp at the fundus. Which of the
following is the most appropriate next step in management?
A. GnRH agonist therapy
B. Medroxyprogesterone acetate therapy
C. Hysteroscopic polypectomy
D. Total vaginal hysterectomy
E. Total abdominal hysterectomy

6. A 40-year-old patient complained of cramping pain in the lower abdomen and


bleeding from the genital tract. A gynecological examination revealed the presence
of myoma node, which is born. Choose the right tactic.
A. Hormonal hemostasis

73
B. Removal of myoma node through the vagina
C. Hysterectomy without appendages
D. Hysterectomy with appendages
E. Supravaginal amputation of the uterus without appendages

7. A 24-year-old woman with signs of peritonitis was urgently admitted to the


hospital. 16 hours ago, she had a criminal abortion. Objectively: the patient is pale,
the tongue is dry. The abdominal wall is not involved in breathing, it is tense on
palpation, Shchetkin-Blumberg symptom is positive in all areas. In the blood:
leukocyte - 15 · 109 / l, neutrophils - 20%. What is the treatment tactic?
A. Urgent laparotomy
B. Diagnostic laparocentesis
C. Diagnostic laparoscopy
D. Conservative treatment, observation
E. Conduct plasmapheresis

8. A 47-year-old patient complains of heavy menstruation. At gynecological


examination: cervix deformed old scars postpartum ruptures. The body of the
uterus is increased accordingly up to 15-16 weeks of pregnancy, dense consistency,
lumpy. What is the most appropriate treatment?
A. Conservative myomectomy
B. Hormone therapy
C. Hemostatic therapy
D. Hysterectomy
E. Supravaginal amputation of the uterus

9. The indication for the operation of Wertheim is:


A. Ovaryan cancer II st.
B. Cervical Cancer I-II.
C. Cervical Cancer III.
D. Cervical cancer IV.
E. Chorionpitelioma

10. In a pregnant woman at the first 28-week examination, cervical cancer in stage
II was detected. Which of the following should be selected?
A. Chemotherapy
B. Combined Radiation Therapy

74
C. Prolongation of pregnancy to the term of delivery
D. Caesarean section with subsequent Wertheim operation
E. Urgent delivery through natural birth lines

11. A 25-year-old patient has surgery on the cyst of the right ovary. During the
operation revealed torsion pedicle of cyst at 720 °. Cyst and spreaded out on her
uterine tube of blue color. What should be the scope of the operation?
A. Right-sided adnexectomy
B. Release the pedicle of the cyst, and then do the right adnexectomy
C. Remove the right appendages and the left fallopian tube
D. Right-sided adnexectomy and removal of salivary
E. Right-sided adnexectomy and appendectomy

12. A woman of 28 years old has been admitted to terminate her pregnancy at 9
weeks. From anamnesis: pregnancies - 5, of them - births - 2, artificial abortions -
3. After the last artificial abortion was treated in connection with
metroendometritis. During the operation, the perforation of the uterus occurred
with a curettage in the area of the bottom of the uterus. What further tactics should
choose?
A. To monitor the patient
B. Complete the removal of the fetal egg residue through the cervical canal
C. Conduct laparotomy and superficial amputation of mats
D. Conduct laparotomy and extirpation of the uterus
E. Perform a laparotomy, examination of the abdominal organs, removal of
the ovum residues from the uterus and suturing the perforation

13. A 54-year-old patient is undergoing surgery for a tumor of the right ovary with
intraoperative identified metastases of the omentum. What should be the scope of
surgery?
A. Biopsy of the omentum
B. Panhysterectomy and removal of the omentum
C. Ovarian biopsy
D. Removal of right uterine appendages and omentum
E. Bilateral adnexectomy and omentum biopsy

14. A 18-year-old patient complains of delayed menstruation for 15 days.

75
Previously, menstrual irregularities were not. Sex life is regular, there were no
pregnancies. On examination: the general condition is satisfactory, the abdomen is
soft, painless. BP - 120/80 mm Hg. Art., pulse - 72 beats. / min. Pregnancy test is
positive. With transvaginal echography diagnosed: progressive ectopic pregnancy.
What is the optimal tactics of a hospital doctor?
A. Hysteroscopy
B. Laparotomy in urgent order
C. Puncture of the abdominal cavity through the posterior fornix
D. Medical diagnostic laparoscopy
E. Laparotomy routinely

76
Bibliography.

1. Адаскевич В.П. «Инфекции, передающиеся половым путем», Москва:


Медкнига, Н. Новгород: НГМА - 2001 г.

2. Акушерская тактика при тазовых предлежаниях, Стрижаков А.Н., Игнатко


И.В., М.: Династия, 2009.

3. Акушерство. Национальное руководство. Гриф УМО по медицинскому


образованию. Айламазян Э.К., Радзинский В.Е., Кулаков В.И., Савельева
Г.М. 2009 г. Издательство: Гэотар-Медиа.

4. Акушерство: курс лекций. Гриф УМО по медицинскому образованию.


Стрижаков А.Н., Давыдов А.И., Буданов П.В., Баев О.Р. 2009 г.
Издательство: Гэотар-Медиа.

5. Анатомически и клинически узкий таз. Чернуха Е.А., Пучко Т.К., Волобуев


А.И. 2005 г. Издательство: Триада-Х.

6. Вихляева Е.М. «Руководство по гинекологической эндокринологии», М:


ООО «Мед. информ. Агентство» - 2000 г.

7. Вопросы патогенеза, морфологической диагностики и клинико-


морфологических сопоставлений. Практическое руководство Цинзерлинг
В.А., Мельникова В.Ф. СПб.: ЭЛБИ, 2002, 344с.

8. Воскресенский С.Л. Оценка состояния плода. Кардиотокография.


Допплерометрия. Биофизический профиль; Учеб. пособие. - Мн.: Книжный
Дом,

9. Гранитов В.М. «Хламидиозы», Москва: Медкнига, Н. Новгород: НГМА -


2000 г.

10.Детская анестезиология и реаниматология Михельсон В.А., Гребенников


В.А. 2-е изд., М. Медицина, 510с.

11.Дубиле П., Бенсон К.Б. Атлас по ультразвуковой диагностике в акушерстве и


гинекологии, 2009 г. Издательство: МЕДпресс-информ.

12.Интенсивная терапия. Анестезиология. Реаниматология. Маневич А.З. 2007г.


М. «Медиздат».

77
13.Клинические рекомендации. Акушерство и гинекология. Савельева Г.М.,
Серов В.Н., Сухих Г.Т. 2009 г. Издательство: Гэотар-Медиа.

14.Клиническое руководство по контрацепции. /Перевод с английского/ Под


редакцией профессора В.Н. Прилепской – 2009 г., М.:Издательство БИНОМ.

15.Неонатология: национальное руководство. Гриф УМО по медицинскому


образованию. Редактор: Володин Н.Н. 2007 г.

16.Неотложные состояния в акушерстве и гинекологии: диагностика и лечение.


Пирлман М., Тинтиналли Дж. 2008 г. Издательство: Бином. Лаборатория
знаний.

17.Оперативное акушерство Манро Керра. Томас Ф., Баскетт Эндрю А Калдер,


Рид Элсивер, М., 2010.

18.Острый ДВС-синдром при критических состояниях в акушерско-


гинекологической клинике: руководство для врачей. Грицан А.И.,
Колесниченко А.П., Грицан Г.В. 2008 г. Издательство: СпецЛит.

19.Руководство по акушерству. Бумм Э. 2007 г. Издательство: МИА

20.Руководство по организации и деятельности перинатального центра. О.В.


Макаров, Н.Н.. Николаев, П.В. Козлов (Под редакцией акад. Володина Н.Н.,
акад. Кулакова В.И., акад. Хальфина Р.А., Шалина Р.И., Дегтярев, Д.Н.,
Мальцева С.А.) - Гэотар –Медиа .- Москва.- 2007.

21.Степанковская Г.К., Венцковский Б.М. «Неотложные состояния в акушерстве


и гинекологии», Киев: Здоровье - 2000 г.
22.Стрижаков А.Н., Давыдов А.И., Белоцерковцева Л.Д. «Клинические лекции
по акушерству и гинекологии», Москва: Медицина - 2000 г.
23.Схемы лечения. Акушерство и гинекология. Под редакцией Кулакова В. И. и
Серова В. Н. 2007 г. Издательство: Литтерра.

24.Течение и ведение беременности по триместрам. Гриф УМО по


медицинскому образованию. Сидорова И.С., Макаров И.О. 2007 г.

25.Тромбозы и тромбоэмболии в акушерско-гинекологической клинике.


Макацария А.Д. 2007 г. Издательство: МИА.

26.Эндокринология беременности в норме и при патологии. Сидельникова В.М.


2009 г. Издательство: МЕДпресс-информ.

78
27.ACOG Practice Bulletin No.142: Cerclage for the management of cervical
insufficiency. American College of Obstetricians and Gynaecologists Obstet
Gynecol. 2014; 123 (2 Pt 1):372.

28.American College of Obstetricіans and Gynaecologists «Hypertension in


pregnancy» . – 2013 y. – 100 p.

29.Antonio Malvasi, Andrea Tinelli, Gian Carlo Di Renzo «Management and Therapy
of Late Pregnancy Complications». - Bari, Italy. - 2017 y. – 396 p.

30.C. Simon Herington «Pathology of cervix». - Edinburg, UK – 2017 y. - 266 р.

31.Cambridge University Press «Basic Practical Skills in Obstetrics and


Gynaecology» . - 2017 y. – 123 p.

32.Cervical intraepithelial neoplasia: Reproductive effects of treatment. Jakobsson M,


Norwitz E R. Up to date May 2014

33.Chan YY, Jayaprakasan K, Tan A, et al. Reproductive outcomes in women with


congenital uterine anomalies: a systematic review. Ultrasound Obstet Gynecol
2011; 38:371.

34.Debbie Saslow, PhD, Diane Solomon, MD, Herschel W. Lawson, MD and


collaborators «American Cancer Society, American Society for Colposcopy and
Cervical Pathology, and American Society for Clinical Pathology Screening
Guidelines for the Prevention and Early Detection of Cervical Cancer» . -
American Society for Colposcopy and Cervical Pathology Journal of Lower
Genital Tract Disease, Volume 16, Number 3, 2012.

35. Diabetes in pregnancy: management Diabetes in pregnancy: management from


preconception to the postnatal from preconception to the postnatal period period. -
NICE guideline Published: 25 February 2015 nice.org.uk/guidance/ng

36.Diagnosis and Management of Cervical Insufficiency GLM0055

37.Helen Bickerstaff MD « Gynaecology 20th e dition by ten Teachers»


MedicalEducation. - King’s College London. – 2017 y. – 399 p.

38.Joelle Borhart «Emergency department management of obstetrics complications».


– Washington, USA . – 2017 y. - 180 р.

39.Johnstone FD, Beard RJ, Boyd IE, McCarthy TG. Cervical diameter after suction
termination of pregnancy. Br Med J 1976; 1:68.

79
40.Leduc L, Wasserstrum N. Successful treatment with the Smith-Hodge pessary of
cervical incompetence due to defective connective tissue in Ehlers-Danlos
syndrome. Am J Perinatol 1992; 9:25.

41.Llewellyn - Jones «Fundamentals of Obstetrics and Gynaecology» - 2017 y. –


387 p.

42.Management» NHSCSP Publication number 20 Third Edition March 2016. -


Public Health England leads the NHS Screening Programmes.

43.NHS Cervical Screening Programme «Colposcopy and Programme

44.OXFORD MEDICAL PUBLICATIONS «Emergencies in Obstetrics and


Gynaecology- 2016 y. – 269 p.

45.Review Date: May 2019 Maternity Guidelines Group, - Christchurch New


Zealand, 04/2017.

46.Romero, R, Lockwood, CJ. Pathogenesis of spontaneous preterm labor. Creasy &


Resnik's Maternal Fetal Medicine, Creasy, RK, Resnik, R, Iams, JD, Lockwood,
CJ, Moore, TR (Eds), Saunders, 2009

47.Sue Pavord, Beverley Hunt «The Obstetrics Hematology Manual». - Cambridge


CB2 8BS, United Kingdom. - 2018 y. – 356 p.

48.Vincenzo Berghella, MD. Cervical insufficiency. Up to date May, 2014.

80

You might also like